You are on page 1of 298

IKI10600I: Discrete Mathematics I

Propositional Logic

Adila A. Krisnadhi
Faculty of Computer Science, University of Indonesia

General Introduction

Propositions

Examples

Propositional Logic
Outline

1 2

General Introduction Propositions The Basics Compound propositions Examples Translating English sentences System specications Boolean searches Logic puzzles Logic and bit operations

General Introduction

Propositions

Examples

What is Discrete Mathematics?

General Introduction

Propositions

Examples

What is Discrete Mathematics?

discrete: consisting of distinct or unconnected elements; opposite of continuous discrete mathematics: mathematics of discrete objects typical topics: logic, arithmetics, algorithms, number theory, counting, set theory, graph theory, etc.

General Introduction

Propositions

Examples

Why study Discrete Mathematics?

General Introduction

Propositions

Examples

Why study Discrete Mathematics?


Because it is needed for answering the following questions: How many ways are there to choose a valid password on a computer system? Is there a link between two computers in a network? How can I identify spam e-mail messages? How can I encyrpt a message so that no unintended recipient can read it? What is the shortest path between two cities using a transportation system? How can a circuit that adds two integers be designed? How many valid Internet addresses are there? ...

General Introduction

Propositions

Examples

List of Topics for Discrete Mathematics I

Logic: propositional, rst order Sets, functions, sequences and summations Number theory Mathematical induction

General Introduction

Propositions

Examples

List of Topics for Discrete Mathematics I

Logic: propositional, rst order Sets, functions, sequences and summations Number theory Mathematical induction

Important emphasis
problem solving skills, especially on reasoning and constructing mathematical proofs.

General Introduction The Basics

Propositions

Examples

Propositions
Denition
proposition: a declarative sentence that has a denite truth value (either true or false, but not both)

General Introduction The Basics

Propositions

Examples

Propositions
Denition
proposition: a declarative sentence that has a denite truth value (either true or false, but not both) Propositions are typically represented using letters: p, q, r , s, p1 , p2 . . . , q1 , q2 , . . .

General Introduction The Basics

Propositions

Examples

Propositions
Denition
proposition: a declarative sentence that has a denite truth value (either true or false, but not both) Propositions are typically represented using letters: p, q, r , s, p1 , p2 . . . , q1 , q2 , . . . Possible truth values for propositions: true and false (you can use T and F ; or 1 and 0, etc.)

General Introduction The Basics

Propositions

Examples

Propositions
Denition
proposition: a declarative sentence that has a denite truth value (either true or false, but not both) Propositions are typically represented using letters: p, q, r , s, p1 , p2 . . . , q1 , q2 , . . . Possible truth values for propositions: true and false (you can use T and F ; or 1 and 0, etc.) An interpretation (truth assignment): mapping that maps propositional variables to their truth values.

General Introduction The Basics

Propositions

Examples

Propositions
Denition
proposition: a declarative sentence that has a denite truth value (either true or false, but not both) Propositions are typically represented using letters: p, q, r , s, p1 , p2 . . . , q1 , q2 , . . . Possible truth values for propositions: true and false (you can use T and F ; or 1 and 0, etc.) An interpretation (truth assignment): mapping that maps propositional variables to their truth values. Example notation: pI = T means p is interpreted as true. Similarly, pI = F means p is false.

General Introduction The Basics

Propositions

Examples

Propositions: examples
Example
Jakarta is the capital of Indonesia. There are 19 new students of this class. The students are on time for todays class.

General Introduction The Basics

Propositions

Examples

Propositions: examples
Example
Jakarta is the capital of Indonesia. There are 19 new students of this class. The students are on time for todays class.

Are the following propositions? Why?


2 + 2 = 4. Study this subject well. x < 2. Blablabla. Do you know more examples of propositions? I know more examples of propositions

General Introduction Compound propositions

Propositions

Examples

Logical connectives
Propositions that you have encountered up to now is called atomic proposition. If you have one or two propositions, you can form a new proposition using a logical connective or logical operator. Such a proposition is called compound proposition.

General Introduction Compound propositions

Propositions

Examples

Logical connectives
Propositions that you have encountered up to now is called atomic proposition. If you have one or two propositions, you can form a new proposition using a logical connective or logical operator. Such a proposition is called compound proposition. There are two kinds of logical operator:
unary operator (needs only one operand): negation () binary operator (needs two operands):
conjunction () disjunction () exclusive-or () implication/conditional () biconditional ().

General Introduction Compound propositions

Propositions

Examples

Negation
Denition
Let p be a proposition. Then p is also a proposition. p is called the negation of p and read as not p.

General Introduction Compound propositions

Propositions

Examples

Negation
Denition
Let p be a proposition. Then p is also a proposition. p is called the negation of p and read as not p.

Important!
p has always the opposite meaning of p. This can be described using the following truth table: p T F p F T

General Introduction Compound propositions

Propositions

Examples

Negation: examples
Example
p: Jakarta is the capital of Indonesia p: Jakarta is not the capital of Indonesia; or It is not the case that Jakarta is the capital of Indonesia q: Today is Friday q: Today is not Friday or It is not the case that today is Friday

General Introduction Compound propositions

Propositions

Examples

Negation: examples
Example
p: Jakarta is the capital of Indonesia p: Jakarta is not the capital of Indonesia; or It is not the case that Jakarta is the capital of Indonesia q: Today is Friday q: Today is not Friday or It is not the case that today is Friday

What is the negation of each of the following?


Jeff never sleeps in class. Jakarta has more than 10 million inhabitants.

General Introduction Compound propositions

Propositions

Examples

Conjunction
Denition
Let p and q be propositions. Then p q is also a proposition. p q is called the conjunction of p and q, read as p and q.

General Introduction Compound propositions

Propositions

Examples

Conjunction
Denition
Let p and q be propositions. Then p q is also a proposition. p q is called the conjunction of p and q, read as p and q.

Important!
p q is true whenever both p and q are true. Otherwise, it is false. p T T F F q T F T F pq T F F F

General Introduction Compound propositions

Propositions

Examples

Conjunction: examples

Example
p: Today is Friday. q: It is raining today. p q: Today is Friday and it is raining today. p: This class is interesting. q: Class participation is lacking. p q: This class is interesting, but class participation is lacking.

General Introduction Compound propositions

Propositions

Examples

Disjunction
Denition
Let p and q be propositions. Then p q is also a proposition. p q is called the disjunction of p and q, read as p or q.

General Introduction Compound propositions

Propositions

Examples

Disjunction
Denition
Let p and q be propositions. Then p q is also a proposition. p q is called the disjunction of p and q, read as p or q.

Important!
p q is true whenever at least one among p and q is true, otherwise it is false. p T T F F q T F T F pq T T T F

General Introduction Compound propositions

Propositions

Examples

Disjunction: examples
Example
p: Today is Friday. q: It is raining today. p q: Today is Friday or it is raining today. p: This job requires experience in web design. q: This job requires experience in web programming. p q: This job requires experience in web design or web programming.

General Introduction Compound propositions

Propositions

Examples

Disjunction: examples
Example
p: Today is Friday. q: It is raining today. p q: Today is Friday or it is raining today. p: This job requires experience in web design. q: This job requires experience in web programming. p q: This job requires experience in web design or web programming.

Important!
The use of or in disjunction is inclusive, i.e., p q is still true whenever both p and q are true. For the exclusive or, see next slide.

General Introduction Compound propositions

Propositions

Examples

Exclusive Or
Denition
Let p and q be propositions. Then p q is also a proposition. p q is called the exclusive or of p and q, read as p xor q.

General Introduction Compound propositions

Propositions

Examples

Exclusive Or
Denition
Let p and q be propositions. Then p q is also a proposition. p q is called the exclusive or of p and q, read as p xor q.

Important!
p q is true whenever exactly one among p and q is true, otherwise it is false. p T T F F q T F T F pq F T T F

General Introduction Compound propositions

Propositions

Examples

Exclusive Or: examples

Example
p: I pass this course. q: I fail this course. p q: (Either) I pass this course or I fail it. p: You can choose computer science. q: You can choose information systems. p q: You can choose (either) computer science or information systems, (but not both).

General Introduction Compound propositions

Propositions

Examples

Implication/Conditional
Denition
Let p and q be propositions. Then p q is also a proposition, called implication or conditional statement. p is called the hypothesis/antecedent/premise; and q is called the conclusion/consequence.

General Introduction Compound propositions

Propositions

Examples

Implication/Conditional
Denition
Let p and q be propositions. Then p q is also a proposition, called implication or conditional statement. p is called the hypothesis/antecedent/premise; and q is called the conclusion/consequence.

p q is read as:
if p, (then) q p is sufcient for q q if p q when p a necessary condition for p is q q unless p p implies q a sufcient condition for q is p p only if q q whenever p q is necessary for p q follows from p

General Introduction Compound propositions

Propositions

Examples

Implication/Conditional
Important!
p q is true whenever both p and q are true; or when p is false. Thus, p q is only false when p is true and q is false. p T T F F q T F T F pq T F T T

General Introduction Compound propositions

Propositions

Examples

Implication: examples

Example
p: Jeff gets seasick. q: Jeff is on a boat. p q: If Jeff gets seasick, then Jeff is on a boat. p: Tuesday is a holiday. q: 2 + 3 = 5. p q: Tuesday is a holiday implies 2 + 3 = 5. Try restate these examples using patterns in the previous slide. When do these implications become true?

General Introduction Compound propositions

Propositions

Examples

Implication: contrapositive, converse and inverse


Denition
Contrapositive of p q is the proposition q p. Converse of p q is the proposition q p. Inverse of p q is the proposition p q.

General Introduction Compound propositions

Propositions

Examples

Implication: contrapositive, converse and inverse


Denition
Contrapositive of p q is the proposition q p. Converse of p q is the proposition q p. Inverse of p q is the proposition p q. What are the contrapositive, converse and inverse of the examples in the previous slide?

General Introduction Compound propositions

Propositions

Examples

Implication: contrapositive, converse and inverse


Denition
Contrapositive of p q is the proposition q p. Converse of p q is the proposition q p. Inverse of p q is the proposition p q. What are the contrapositive, converse and inverse of the examples in the previous slide? Given a conditional statement, which one is equivalent (always has the same truth value) to it? Its contrapositive, converse or inverse?

General Introduction Compound propositions

Propositions

Examples

Implication: contrapositive, converse and inverse


Denition
Contrapositive of p q is the proposition q p. Converse of p q is the proposition q p. Inverse of p q is the proposition p q. What are the contrapositive, converse and inverse of the examples in the previous slide? Given a conditional statement, which one is equivalent (always has the same truth value) to it? Its contrapositive, converse or inverse? Prove it.

General Introduction Compound propositions

Propositions

Examples

Contrapositive, converse, inverse: truth table

contrapositive

converse

inverse

p T T F F

q T F T F

p F F T T

q F T F T

pq T F T T

q p T F T T

qp T T F T

p q T T F T

General Introduction Compound propositions

Propositions

Examples

Biconditional/bi-implication/equivalence

Denition
Let p and q be propositions. Then p q is also a proposition, called bi-implication or biconditional statement.

General Introduction Compound propositions

Propositions

Examples

Biconditional/bi-implication/equivalence

Denition
Let p and q be propositions. Then p q is also a proposition, called bi-implication or biconditional statement.

p q is read as:
p if and only if q if p then q, and conversely. p is equivalent with q p iff q p is necessary and sufcient for q. p and q are equivalent

General Introduction Compound propositions

Propositions

Examples

Biconditional/Bi-implication: truth value


Important!
p q is true exactly when p and q have the same truth value. p q is true when both p q and q p is true. See truth table below. p T T F F q T F T F pq T F T T qp T T F T (p q) (q p) T F F T pq T F F T

General Introduction Compound propositions

Propositions

Examples

Biconditional: examples

Example
p: You can take the ight. q: You buy a ticket. p q: You can take the ight if and only if you buy a ticket. p: For you to win the contest. q: You have the only winning ticket. p q: For you to win the contest, it is necessary and sufcient that you have the only winning ticket.

General Introduction Compound propositions

Propositions

Examples

Truth tables of all logical connectives

negation

conjunction

disjunction

exclusive or

implication

bi-implication

NOT

AND

OR

XOR

IMPLIES

IFF

p T T F F

q T F T F

p F F T T

pq T F F F

pq T T T F

pq F T T F

pq T F T T

pq T F F T

General Introduction Compound propositions

Propositions

Examples

Precedence of logical connectives


Operator Precedence 1 2 3 4 5

Use parantheses whenever necessary!

Example
p q r means (p q) r p q means (p) q p q r means (p q) r

General Introduction Translating English sentences

Propositions

Examples

Translating English sentences

Translate the following sentence into logical expression: You cannot ride the roller coaster if you are under 130 cm tall unless you are older than 16 years old.

General Introduction Translating English sentences

Propositions

Examples

Translating English sentences

Translate the following sentence into logical expression: You cannot ride the roller coaster if you are under 130 cm tall unless you are older than 16 years old.

Answer
Let p: You can ride the roller coaster, q: You are under 130 cm tall, and r : You are older than 16 years old. Then the sentence can be translated into (q r ) p

General Introduction System specications

Propositions

Examples

System specications are expected to be consistent, i.e., they should not contain conicting requirements that can be used to derive a contradiction.

General Introduction System specications

Propositions

Examples

System specications are expected to be consistent, i.e., they should not contain conicting requirements that can be used to derive a contradiction.

Are the following specications consistent?


The system is in multiuser state if and only if it is operating normally. If the system is operating normally, the kernel is functioning. The kernel is not functioning or the system is in interrupt mode. The system is not in interrupt mode.

General Introduction System specications

Propositions

Examples

System specications are expected to be consistent, i.e., they should not contain conicting requirements that can be used to derive a contradiction.

Are the following specications consistent?


The system is in multiuser state if and only if it is operating normally. If the system is operating normally, the kernel is functioning. The kernel is not functioning or the system is in interrupt mode. The system is not in interrupt mode.

Answer
Suppose p: The system is in multiuser state, q: The system is operating normally, r : The kernel is functioning and s: The system is in interrupt mode. Thus the specications can be written as: p q, q r , r s, s. We want to nd a truth assignment for p, q, r , s that makes all statements true. First, s must be false to make s true. Thus, to make r s true, r must be false. This means, q must also be false in order to make q r true. Finally, p should be false to make p q true. Hence, the specications are consistent because they are all true when p, q, r , s are all false. (Check this using truth table).

General Introduction Boolean searches

Propositions

Examples

Boolean searches
Logical connectives are used for searching in large collections of information. E.g., in search engine. AND: match records containing both two search terms OR: match records containing at least one of the two search terms NOT: exclude records containing the search term.

Example
We can search pages that contains the term universitas, either the term Indonesia or Bogor, but does not contain the term Semarang. search term: UNIVERSITAS AND (INDONESIA OR BOGOR) AND NOT SEMARANG in Google: universitas indonesia OR bogor -semarang

General Introduction Logic puzzles

Propositions

Examples

Inhabitants of an island can be divided into the knights and the knaves. Knights always tell the truth and knaves always lie. Suppose you meet two inhabitants of this island, say A and B. A says At least one of us is a knave and B says nothing. Can you tell whos the knight and/or whos the knave?

General Introduction Logic puzzles

Propositions

Examples

Inhabitants of an island can be divided into the knights and the knaves. Knights always tell the truth and knaves always lie. Suppose you meet two inhabitants of this island, say A and B. A says At least one of us is a knave and B says nothing. Can you tell whos the knight and/or whos the knave?

Answer
Let p: A is a knight and q: B is a knight, so that p means A is a knave and q means B is a knave. We can thus express As statement as p q.

General Introduction Logic puzzles

Propositions

Examples

Inhabitants of an island can be divided into the knights and the knaves. Knights always tell the truth and knaves always lie. Suppose you meet two inhabitants of this island, say A and B. A says At least one of us is a knave and B says nothing. Can you tell whos the knight and/or whos the knave?

Answer
Let p: A is a knight and q: B is a knight, so that p means A is a knave and q means B is a knave. We can thus express As statement as p q. First suppose p is true, i.e., A is a knight. Then As statement, i.e., p q is true. It follows that q must be true, i.e., B is a knave.

General Introduction Logic puzzles

Propositions

Examples

Inhabitants of an island can be divided into the knights and the knaves. Knights always tell the truth and knaves always lie. Suppose you meet two inhabitants of this island, say A and B. A says At least one of us is a knave and B says nothing. Can you tell whos the knight and/or whos the knave?

Answer
Let p: A is a knight and q: B is a knight, so that p means A is a knave and q means B is a knave. We can thus express As statement as p q. First suppose p is true, i.e., A is a knight. Then As statement, i.e., p q is true. It follows that q must be true, i.e., B is a knave. To ensure that this is the only answer, lets take a look what happen if we assume p as false. If p is false, then p is true, i.e., A must be a knave. This means As statement, p q must be false. But to make p q false, we require p to be false too which is impossible since we already have that p is true.

General Introduction Logic puzzles

Propositions

Examples

Inhabitants of an island can be divided into the knights and the knaves. Knights always tell the truth and knaves always lie. Suppose you meet two inhabitants of this island, say A and B. A says At least one of us is a knave and B says nothing. Can you tell whos the knight and/or whos the knave?

Answer
Let p: A is a knight and q: B is a knight, so that p means A is a knave and q means B is a knave. We can thus express As statement as p q. First suppose p is true, i.e., A is a knight. Then As statement, i.e., p q is true. It follows that q must be true, i.e., B is a knave. To ensure that this is the only answer, lets take a look what happen if we assume p as false. If p is false, then p is true, i.e., A must be a knave. This means As statement, p q must be false. But to make p q false, we require p to be false too which is impossible since we already have that p is true. Hence, we conclude that A is a knight and B is a knave.

General Introduction Logic and bit operations

Propositions

Examples

Bit operation

Bit (Binary digit): a symbol with two possible values (0 and 1). Computers represent information using bits. Bit can be used to represent truth value: 1 represents true and 0 represents false. Bit operations: analogous to operations using logical connectives (AND, OR, XOR). Bit string: sequence of zero or more bits. Its length is the number of bits in the string. Example: 101010011 is a bit string of length nine.

General Introduction Logic and bit operations

Propositions

Examples

Bitwise AND, bitwise OR, bitwise XOR


Find the bitwise AND, bitwise OR and bitwise XOR of the bit strings 1011 0011 and 1110 0001.

General Introduction Logic and bit operations

Propositions

Examples

Bitwise AND, bitwise OR, bitwise XOR


Find the bitwise AND, bitwise OR and bitwise XOR of the bit strings 1011 0011 and 1110 0001.

Answer
1011 0011 1110 0001 1010 0001 b i t w i s e AND 1111 0011 b i t w i s e OR 0101 0010 b i t w i s e XOR

IKI10600I: Discrete Mathematics I


Logical Equivalence

Adila A. Krisnadhi
Faculty of Computer Science, University of Indonesia

Interpretation

Propositional Equivalence

Logical Equivalence
Outline

Interpretation

Propositional Equivalence

Interpretation

Propositional Equivalence

Interpretation
Denition
Interpretation is a truth assignment to a (possibly compound) proposition. Interpretation can be described simply by truth assignments of all propositional variables of the proposition. Truth assignment of a compound proposition can then be computed from truth assignments of its propositional variables. Each row of the truth table of a proposition corresponds to a particular interpretation of the proposition.

Interpretation

Propositional Equivalence

Truth tables of compound proposition

Construct the truth table for (p q) (p q)


Interpretations I1 : p I2 : p I3 : p I4 : p
I1 I2 I3 I4

q F T F T

p q T T F T

pq T F F F

(p q) (p q) T F T F

= T, q = T, q = F, q = F, q

I1 I2 I3 I4

=T =F =T =F

Interpretation

Propositional Equivalence

Truth tables of compound proposition


Construct the truth table for (p q) (p q) p T T F F q T F T F q F T F T p q T T F T pq T F F F (p q) (p q) T F T F

The compound proposition above used 2 variables. Its truth table contains 4 rows. How many rows are there in the truth table of a compound proposition that uses 3 variables? 4 variables? n variables?

Interpretation

Propositional Equivalence

Validity, satisability and contradiction


Denition
Let G be a proposition. G is valid iff G is true for every interpretation of G. In this case, G is also called a tautology. G is satisable iff there exists at least one interpretation of G for which G is true. G is falsiable iff there exists at least one interpretation of G for which G is false. G is unsatisable/contradictory iff G is false for every interpretation of G. In this case, G is also called a contradiction. Note: A proposition that is both not valid and non contradictory is also called contingency.

Interpretation

Propositional Equivalence

Equivalence

Denition
Let p and q be two propositions. p is equivalent with q iff the proposition p q is a tautology. In other words, p and q are equivalent iff p and q have the same truth value on every row of their truth tables. Instead of using , we often use to denote equivalence.

Interpretation

Propositional Equivalence

Proving equivalence

If you are given two propositions p and q, how can you show that they are equivalent? There are two ways of proving two propositions equivalent:
using truth tables; using laws of equivalence.

Interpretation

Propositional Equivalence

Proving equivalence using truth table

Show that p q and (p q) are equivalent (i.e., p q (p q) is a tautology). p T T F F q T F T F pq T T T F p F F T T q F T F T p q F F F T (p q) T T T F G T T T T

Note: we dene G := p q (p q)

Interpretation

Propositional Equivalence

Proving equivalence using truth table

Show that p q and p q are equivalent. p T T F F q T F T F pq T F T T p F F T T p q T F T T

Since p q and p q have the same truth values on each row of the truth table, both of them are equivalent.

Interpretation

Propositional Equivalence

Some logical equivalences (1)

pTp pFp pTT pFF pp p pp p (p) pq qp pq qp

Identity laws Domination laws Idempotent laws Double negation laws Commutative laws

Interpretation

Propositional Equivalence

Some logical equivalences (2)

(p q) r p (q r ) (p q) r p (q r ) p (q r ) (p q) (p r ) p (q r ) (p q) (p r ) (p q) p q (p q) p q p (p q) p p (p q) p p p T p p F

Associative laws Distributive laws De Morgans laws Absorption laws Negation laws

Interpretation

Propositional Equivalence

Some logical equivalences (3)


p q p q p q q p (p q) p q (p q) (p r ) p (q r ) (p r ) (q r ) (p q) r (p q) (p r ) p (q r ) (p r ) (q r ) (p q) r p q (p q) (q p) p q p q p q (p q) (p q) (p q) p q

Interpretation

Propositional Equivalence

Proving equivalence without truth table


Show that (p (p q)) and p q are equivalent

Answer
(p (p q)) p (p q) p ((p) q) p (p q) by De Morgan laws by De Morgan laws by double negation law (p p) (p q) by distributivity F (p q) (p q) F p q because p p F by commutativity of disjunction by identity law for F

Interpretation

Propositional Equivalence

Proving tautology without truth table


Show that (p q) (p q) is a tautology.

Answer
We show that the statement is equivalent to T. (p q) (p q) (p q) (p q) since p q p q (p q) (p q) by De Morgan laws (p p) (q q) by associativity and
commutativity of

TT T

by commutativity of and since p p T by domination law

IKI10600I: Discrete Mathematics I


Logical Equivalence: continued

Adila A. Krisnadhi
Faculty of Computer Science, University of Indonesia

Logical Equivalence: continued


Outline

Proving tautology without truth table


Show that (p q) (p q) is a tautology.

Answer
To show that the statement is valid, we assume that there exists an interpretation making it false, and derive a contradiction. Assume that (p q) (p q) is false. It must be the case that (p q) is true and (p q) is false. Since (p q) is false, p q must be true. Since p q is true, we have both p and q are true. p that p and q are true implies that both p and q are false. Since p and q are false, (p q) must be false. Since we already have that (p q) is true, we obtain a contradiction. Therefore, our assumption that (p q) (p q) is false is incorrect, i.e., (p q) (p q) is always true.

More examples

Determine whether the following statements are valid, or contradictory or neither. (p q) (p q) (p q) (p q) p (p q) q ((p q) (q r )) (p r ) (p q) (p q)

Determining validity of a proposition: summary

Use a truth table:


valid if the proposition is true on each row of the truth table; straightforward; difcult if there are too many variables (2n rows for n variables)

Use logical equivalence laws Use a contradiction:


Assume that the proposition is false Derive a contradictory truth assignment for the variables/subpropositions Due to contradiction, the initial assumption is incorrect, i.e., the proposition is always true.

IKI10600I: Discrete Mathematics I


Predicates and Quantiers Adila A. Krisnadhi
Faculty of Computer Science, University of Indonesia

Predicates

Quantiers

Predicates and Quantiers


Outline

Predicates

Quantiers Universal quantier Existential quantier More examples and remarks

Predicates

Quantiers

Predicates
The statement x > 3 or x is greater than 3 consists of two parts:
variable x whose possible values are taken from a certain set D predicate is greater than 3

The set D is called the domain or the universe of discourse. We can denote such statement as P(x) where P denotes the predicate and x is the variable. P(x) has no truth value until x is assigned to a certain element from D. A predicate is always associated to a certain arity: the number of variables associated to P An n-ary predicate is a predicate with arity n.

Predicates

Quantiers

Let P(x) be the statement x > 3. What are the truth values of P(5) and P(1)?
P(5) is obtained by setting x = 5 in the statement. Hence, P(5) is true since 5 > 3 is true. P(1) is false since 1 > 3 is false. P is a unary predicate, i.e., a predicate with arity 1.

Let A(c, n) be Computer c is connected to network n where c represents a computer and n represents a network.
A(c, n) is true whenever c is a computer that is connected to the network n, otherwise it is false. A is a binary predicate, i.e., its arity is 2.

Let R(x, y , z) be x + y = z.
R is a ternary predicate, i.e., its arity is 3. R(1, 2, 3) is true whereas R(0, 0, 1) is false.

Predicates

Quantiers

Variable Binding

Variable x in P(x) is bound if x is replaced with an element of the domain D or bound by a quantier. Otherwise it is a free variable. Two mainly used quantiers are universal quantiers and existential quantier.

Predicates Universal quantier

Quantiers

Universal quantier
Denition
The universal quantication of P(x) (xP(x)) is the proposition P(x) is true for all (every) values of x in the domain D is the universal quantier and read for all, for every, for each, for any, for arbitrary, all of, given any, etc. An element for which P(x) is false is called counterexample of xP(x) xP(x) is true when P(x) is true for every x xP(x) is false when there is an x for which P(x) is false. In xP(x, y ), x is a bound variable whereas y is a free variable.

Predicates Universal quantier

Quantiers

Universal Quantier: examples

Let P(x) be x + 1 > x. What is the truth value of xP(x) with R as the domain ? Let Q(x) be x < 2. What is the truth value of xQ(x) with R as the domain? What is the truth value of xP(x), where P(x) denotes x 2 < 10 and the domain consists of positive integers not exceeding 4? Note: for a nite domain where x1 , . . . , xn are its elements, xP(x) is equivalent with P(x1 ) P(x2 ) P(xn )

Predicates Universal quantier

Quantiers

Universal Quantier: examples


Every student in this class attends the course of Foundations of Programming

Predicates Universal quantier

Quantiers

Universal Quantier: examples


Every student in this class attends the course of Foundations of Programming First alternative:
Take D := {x | x is a student in this class} P(x) := x attends the course of Foundations of Programming Then the statement can be written as xP(x)

Predicates Universal quantier

Quantiers

Universal Quantier: examples


Every student in this class attends the course of Foundations of Programming First alternative:
Take D := {x | x is a student in this class} P(x) := x attends the course of Foundations of Programming Then the statement can be written as xP(x)

Second alternative:
Take as the domain: D := {x | x is a student} Q(x) := x is in this class. P(x) := x attends the course of Foundations of Programming Then the statement can be written as x(Q(x) P(x))

Predicates Existential quantier

Quantiers

Existential quantier
Denition
The existential quantication of P(x) (xP(x)) is the proposition P(x) is true for some values of x in the domain D is the existential quantier and read: for some, there exists, there is, at least one, etc. xP(x) is true when there is at least one element x of D for which P(x) is true. xP(x) is false when P(x) is false for every x, i.e., no element xof D for which P(x) is true. In xP(x, y ), x is a bound variable whereas y is a free variable.

Predicates Existential quantier

Quantiers

Existential Quantier: examples

Let P(x) be x > 34. What is the truth value of xP(x) with R as the domain? Let Q(x) be x = x + 1. What is the truth value of xQ(x) with R as the domain? What is the truth value of xP(x), where P(x) denotes x 2 < 10 and the domain consists of positive integers not exceeding 4? Note: for a nite domain where x1 , . . . , xn are its elements, xP(x) is equivalent with P(x1 ) P(x2 ) P(xn )

Predicates More examples and remarks

Quantiers

More examples
Everyone has a close friend

Predicates More examples and remarks

Quantiers

More examples
Everyone has a close friend Because friendship is a relation between two person, we need two variables: x and y

Predicates More examples and remarks

Quantiers

More examples
Everyone has a close friend Because friendship is a relation between two person, we need two variables: x and y Take D := {x | x is a person} as the domain of both variables x and y .

Predicates More examples and remarks

Quantiers

More examples
Everyone has a close friend Because friendship is a relation between two person, we need two variables: x and y Take D := {x | x is a person} as the domain of both variables x and y . Take a predicate A(x, y ) := y is a close friend of x

Predicates More examples and remarks

Quantiers

More examples
Everyone has a close friend Because friendship is a relation between two person, we need two variables: x and y Take D := {x | x is a person} as the domain of both variables x and y . Take a predicate A(x, y ) := y is a close friend of x The sentence becomes: x D(y DA(x, y )) or x(yA(x, y )) or xyA(x, y )

Predicates More examples and remarks

Quantiers

More examples
Everyone has a close friend Because friendship is a relation between two person, we need two variables: x and y Take D := {x | x is a person} as the domain of both variables x and y . Take a predicate A(x, y ) := y is a close friend of x The sentence becomes: x D(y DA(x, y )) or x(yA(x, y )) or xyA(x, y ) The scope for y is A(x, y ) in which x is still a free variable. The scope for x is yA(x, y ) in which x is no longer a free variable.

Predicates More examples and remarks

Quantiers

More examples

Change the following sentences into expressions in predicate logic. Jackie has exactly one close friend. Everyone has exactly one close friend. If someone is a woman and has a child, then she is a mother.

Predicates More examples and remarks

Quantiers

Summary
xP(x) :
is true iff P(x) is true for every x D; is false iff P(x) is false for some x D.

xP(x):
is true iff P(x) is true for some x D; is false iff P(x) is false for every x D.

xyP(x, y ):
is true iff P(x, y ) is true for every pair x Dx and y Dy is false iff P(x, y ) is false for some pair x Dx and y Dy

y xP(x, y ):
is true iff P(x, y ) is true for every pair y Dy and x Dx is false iff P(x, y ) is false for some pair y Dy and x Dx

Predicates More examples and remarks

Quantiers

Summary (2)

xyP(x, y ):
is true iff for every x Dx , there is one y Dy for which P(x, y ) is true; is false iff there is one x Dx such that P(x, y ) is false for every y Dy .

xyP(x, y ):
is true iff there is one x Dx such that P(x, y ) is true for every y Dy ; is false iff for every x Dx , there is one y Dy for which P(x, y ) is false.

Predicates More examples and remarks

Quantiers

Summary (3)

xyP(x, y ):
is true iff there is (at least) one pair x Dx and y Dy such that P(x, y ) is true; is false iff for every pair x Dx and y Dy , P(x, y ) is false.

y xP(x, y ):
is true iff there is (at least) one pair y Dx and x Dy such that P(x, y ) is true; is false iff for every pair y Dx and x Dy , P(x, y ) is false.

Predicates More examples and remarks

Quantiers

More examples

Here the domain of all variables are real numbers. Let Q(x, y ) be the relation x + y = y + x. What is the truth value of xyQ(x, y )? Let Q(x, y ) be the relation x + y = 0. What is the truth value of xyQ(x, y ) and y xQ(x, y )? Let Q(x, y , z) be the relation x + y = z. What is the truth value of xy zQ(x, y , z) and zxyQ(x, y , z)?

Predicates More examples and remarks

Quantiers

Some remarks

xyP(x, y ) y xP(x, y ) xyP(x, y ) y xP(x, y ) xP(x) xP(x) (De Morgans) xP(x) xP(x) (De Morgans) xyP(x, y ) is not equivalent with y xP(x, y ) If the domain for x is empty, the statement:
xP(x) is true, because we cannot nd any counterexample for P(x) (that makes P(x) false); xP(x) is false, because we cannot nd any x in the domain that makes P(x) true.

IKI10600I: Discrete Mathematics I


Rules of Inference Adila A. Krisnadhi
Faculty of Computer Science, University of Indonesia

Valid arguments

Fallacies

Rules of inference for quantied statements

Rules of Inference
Outline
8

Valid arguments Modus Ponens Modus tollens Syllogisms Other rules of inference Resolution Examples Fallacies Rules of inference for quantied statements Rules on universal quantiers Rules on existential quantiers Examples Universal modus ponens & modus tollens

9 10

Valid arguments

Fallacies

Rules of inference for quantied statements

Argument in Logic

Denition
Argument: a sequence of propositions All but the nal proposition is called premises, whereas the nal proposition is the conclusion. An argument is valid if the truth of all its premises implies that the conclusion is true.

Valid arguments Modus Ponens

Fallacies

Rules of inference for quantied statements

Modus Ponens (Law of Detachment)

Modus ponens
p pq q

Valid arguments Modus Ponens

Fallacies

Rules of inference for quantied statements

Modus Ponens (Law of Detachment)

Modus ponens
p pq q
I f you do a l l t h e e x e r c i s e s , then you w i l l suceed i n exams . You do a l l t h e e x e r c i s e s . You w i l l succeed i n exams .

Valid arguments Modus tollens

Fallacies

Rules of inference for quantied statements

Modus Tollens

Modus tollens
q pq p

Valid arguments Modus tollens

Fallacies

Rules of inference for quantied statements

Modus Tollens

Modus tollens
q pq p
I f you do a l l t h e e x e r c i s e s , then you w i l l suceed i n exams . You d i d n o t succeed i n exams . You d i d n o t do a l l t h e e x e r c i s e s .

Valid arguments Syllogisms

Fallacies

Rules of inference for quantied statements

Syllogisms
Hypothetical syllogism
pq qr pr

Valid arguments Syllogisms

Fallacies

Rules of inference for quantied statements

Syllogisms
Hypothetical syllogism
pq qr pr

Disjunctive syllogism
pq p q

Valid arguments Other rules of inference

Fallacies

Rules of inference for quantied statements

Other rules
Addition
p pq

Valid arguments Other rules of inference

Fallacies

Rules of inference for quantied statements

Other rules
Addition
p pq

Simplication
pq p

Valid arguments Other rules of inference

Fallacies

Rules of inference for quantied statements

Other rules
Addition
p pq

Simplication
pq p

Conjunction
p q pq

Valid arguments Resolution

Fallacies

Rules of inference for quantied statements

Resolution
pq p r qr

Valid arguments Resolution

Fallacies

Rules of inference for quantied statements

Resolution
pq p r qr Resolution is the rule of inference for computer to do an automatic reasoning. In the above denition, q r is called the resolvent. In using resolution, hypotheses and conclusion must be expressed as clauses. Clause: disjunction of variables or negations of variables.

Valid arguments Examples

Fallacies

Rules of inference for quantied statements

Is the following argument valid?


Is the following argument valid?
3 If 2 > 3 , then ( 2)2 > ( 3 )2 . We know that 2 > 2 . 2 2 2 Consequently, ( 2) = 2 > ( 3 )2 = 9 . 2 4

Valid arguments Examples

Fallacies

Rules of inference for quantied statements

Is the following argument valid?


Is the following argument valid?
3 If 2 > 3 , then ( 2)2 > ( 3 )2 . We know that 2 > 2 . 2 2 2 Consequently, ( 2) = 2 > ( 3 )2 = 9 . 2 4

Answer

3 Let p be the proposition 2 > 2 and q be the proposition 2 > 3 . The premises of the argument are p q and p, 2 whereas the conclusion is q. Thus, the argument is valid because it is constructed by using modus ponens, a valid argument form.

Valid arguments Examples

Fallacies

Rules of inference for quantied statements

Is the following argument valid?


Is the following argument valid?
3 If 2 > 3 , then ( 2)2 > ( 3 )2 . We know that 2 > 2 . 2 2 2 Consequently, ( 2) = 2 > ( 3 )2 = 9 . 2 4

Answer

3 Let p be the proposition 2 > 2 and q be the proposition 2 > 3 . The premises of the argument are p q and p, 2 whereas the conclusion is q. Thus, the argument is valid because it is constructed by using modus ponens, a valid argument form. However, one of its premises, 2 > 3 , is false. Consequently, 2 we cannot conclude that the conclusion is true. Moreover, note that the conclusion itself is false, because 2 < 9 4

Valid arguments Examples

Fallacies

Rules of inference for quantied statements

Show that the hypotheses If you send me an e-mail, then I will nish writing the program, If you do not send me an e-mail, then I will go to sleep early, and If I go to sleep early, then I will wake up feeling refreshed lead to the conclusion If I do not nish writing the program, then I will wake up feeling refreshed.

Valid arguments Examples

Fallacies

Rules of inference for quantied statements

Show that the hypotheses If you send me an e-mail, then I will nish writing the program, If you do not send me an e-mail, then I will go to sleep early, and If I go to sleep early, then I will wake up feeling refreshed lead to the conclusion If I do not nish writing the program, then I will wake up feeling refreshed.

Solution
Let p be the proposition You send me an e-mail q, the proposition I will nish writing the program r , the proposition I will go to sleep early s, the proposition I will wake up feeling refreshed

Valid arguments Examples

Fallacies

Rules of inference for quantied statements

Show that the hypotheses If you send me an e-mail, then I will nish writing the program, If you do not send me an e-mail, then I will go to sleep early, and If I go to sleep early, then I will wake up feeling refreshed lead to the conclusion If I do not nish writing the program, then I will wake up feeling refreshed.

Solution
Let p be the proposition You send me an e-mail q, the proposition I will nish writing the program r , the proposition I will go to sleep early s, the proposition I will wake up feeling refreshed The hypotheses are p q, p r , r s. The desired conclusion is q s

Valid arguments Examples

Fallacies

Rules of inference for quantied statements

Show that the hypotheses If you send me an e-mail, then I will nish writing the program, If you do not send me an e-mail, then I will go to sleep early, and If I go to sleep early, then I will wake up feeling refreshed lead to the conclusion If I do not nish writing the program, then I will wake up feeling refreshed.

Solution
Let p be the proposition You send me an e-mail q, the proposition I will nish writing the program r , the proposition I will go to sleep early s, the proposition I will wake up feeling refreshed The hypotheses are p q, p r , r s. The desired conclusion is q s
1

pq

Hypothesis

p r

Hypothesis

5 6

r s q s

Hypothesis

Valid arguments Examples

Fallacies

Rules of inference for quantied statements

Show that the hypotheses If you send me an e-mail, then I will nish writing the program, If you do not send me an e-mail, then I will go to sleep early, and If I go to sleep early, then I will wake up feeling refreshed lead to the conclusion If I do not nish writing the program, then I will wake up feeling refreshed.

Solution
Let p be the proposition You send me an e-mail q, the proposition I will nish writing the program r , the proposition I will go to sleep early s, the proposition I will wake up feeling refreshed The hypotheses are p q, p r , r s. The desired conclusion is q s
1 2 3

pq q p p r

Hypothesis Contrapositive of (1) Hypothesis

5 6

r s q s

Hypothesis

Valid arguments Examples

Fallacies

Rules of inference for quantied statements

Show that the hypotheses If you send me an e-mail, then I will nish writing the program, If you do not send me an e-mail, then I will go to sleep early, and If I go to sleep early, then I will wake up feeling refreshed lead to the conclusion If I do not nish writing the program, then I will wake up feeling refreshed.

Solution
Let p be the proposition You send me an e-mail q, the proposition I will nish writing the program r , the proposition I will go to sleep early s, the proposition I will wake up feeling refreshed The hypotheses are p q, p r , r s. The desired conclusion is q s
1 2 3 4 5 6

pq q p p r q r r s q s

Hypothesis Contrapositive of (1) Hypothesis Hypothetical syllogism using (2) and (3) Hypothesis

Valid arguments Examples

Fallacies

Rules of inference for quantied statements

Show that the hypotheses If you send me an e-mail, then I will nish writing the program, If you do not send me an e-mail, then I will go to sleep early, and If I go to sleep early, then I will wake up feeling refreshed lead to the conclusion If I do not nish writing the program, then I will wake up feeling refreshed.

Solution
Let p be the proposition You send me an e-mail q, the proposition I will nish writing the program r , the proposition I will go to sleep early s, the proposition I will wake up feeling refreshed The hypotheses are p q, p r , r s. The desired conclusion is q s
1 2 3 4 5 6

pq q p p r q r r s q s

Hypothesis Contrapositive of (1) Hypothesis Hypothetical syllogism using (2) and (3) Hypothesis Hypothetical syllogism using (4) and (5)

Valid arguments Examples

Fallacies

Rules of inference for quantied statements

Show that the hypotheses (p q) r and r s imply the conclusion p s

Valid arguments Examples

Fallacies

Rules of inference for quantied statements

Show that the hypotheses (p q) r and r s imply the conclusion p s

Solution
(p q) r can be rewritten as two clauses, p r and q r . r s can also be rewritten as r s. Using resolution on the two clauses, p r and r s, we can obtain the conclusion p s.

Valid arguments

Fallacies

Rules of inference for quantied statements

Fallacy of afrming conclusion

This is NOT a valid argument


If you do every problem in this book, then you will learn discrete mathematics. You learned discrete mathematics. Therefore, you did every problem in this book.

Valid arguments

Fallacies

Rules of inference for quantied statements

Fallacy of afrming conclusion

This is NOT a valid argument


If you do every problem in this book, then you will learn discrete mathematics. You learned discrete mathematics. Therefore, you did every problem in this book. This argument is not valid because the proposition [(p q) q] p is NOT a tautology.

Valid arguments

Fallacies

Rules of inference for quantied statements

Fallacy of denying the hypothesis

This is NOT a valid argument


If you do every problem in this book, then you will learn discrete mathematics. You did not do every problem in this book. Therefore, you did not learn discrete mathematics.

Valid arguments

Fallacies

Rules of inference for quantied statements

Fallacy of denying the hypothesis

This is NOT a valid argument


If you do every problem in this book, then you will learn discrete mathematics. You did not do every problem in this book. Therefore, you did not learn discrete mathematics. This argument is not valid because the proposition [(p q) p] q is NOT a tautology.

Valid arguments Rules on universal quantiers

Fallacies

Rules of inference for quantied statements

Universal instantiation

Universal instantiation
x.P(x) P(c)

Valid arguments Rules on universal quantiers

Fallacies

Rules of inference for quantied statements

Universal instantiation

Universal instantiation
x.P(x) P(c) c is a particular member of the domain, given the premise x.P(x) Example: from the statement All women are wise, we can conclude Lisa is wise, provided that Lisa is a member of the domain of all women.

Valid arguments Rules on universal quantiers

Fallacies

Rules of inference for quantied statements

Universal generalization
Universal generalization
P(c) for an arbitrary c x.P(x)

Valid arguments Rules on universal quantiers

Fallacies

Rules of inference for quantied statements

Universal generalization
Universal generalization
P(c) for an arbitrary c x.P(x) Universal generalization is used when we show that x.P(x) is true by taking an arbitrary element c from the domain, and showing that P(c) is true. Arbitrary means that we have no control over c and cannot make any assumptions about c other than it comes from the domain. This rule is often used implicitly in mathematics. Also often used incorrectly, due to making unwarranted assumptions about the arbitrary element c.

Valid arguments Rules on existential quantiers

Fallacies

Rules of inference for quantied statements

Existential instantiation
Existential instantiation
x.P(x) P(c) for some element c

Valid arguments Rules on existential quantiers

Fallacies

Rules of inference for quantied statements

Existential instantiation
Existential instantiation
x.P(x) P(c) for some element c This rule allows us to conclude that there is an element c in the domain for which P(c) is true, if we know that x.P(x) is true. c is NOT arbitrary, but rather it must the c for which P(c) is true. Usually we have no knowledge over c, other that it exists. Because it exists, we may give it a name (c) and continue our argument.

Valid arguments Rules on existential quantiers

Fallacies

Rules of inference for quantied statements

Existential generalization

Existential generalization
P(c) for some element c x.P(x)

Valid arguments Rules on existential quantiers

Fallacies

Rules of inference for quantied statements

Existential generalization

Existential generalization
P(c) for some element c x.P(x) This rule is used to conclude that x.P(x) is true when we know a particular element c with P(c) true.

Valid arguments Examples

Fallacies

Rules of inference for quantied statements

Show that the premises Everyone in this discrete mathematics class also takes a course in foundations of programming, and Eva is a student is this class imply the conclusion Eva takes a course in foundations of programming.

Valid arguments Examples

Fallacies

Rules of inference for quantied statements

Show that the premises Everyone in this discrete mathematics class also takes a course in foundations of programming, and Eva is a student is this class imply the conclusion Eva takes a course in foundations of programming.

Solution
Let D(x) denote x is in this discrete mathematics class, and C(x) denote x takes a course in foundations of programming.

Valid arguments Examples

Fallacies

Rules of inference for quantied statements

Show that the premises Everyone in this discrete mathematics class also takes a course in foundations of programming, and Eva is a student is this class imply the conclusion Eva takes a course in foundations of programming.

Solution
Let D(x) denote x is in this discrete mathematics class, and C(x) denote x takes a course in foundations of programming. The hypotheses are x.(D(x) C(x)) and D(Eva). The desired conclusion is C(Eva)

Valid arguments Examples

Fallacies

Rules of inference for quantied statements

Show that the premises Everyone in this discrete mathematics class also takes a course in foundations of programming, and Eva is a student is this class imply the conclusion Eva takes a course in foundations of programming.

Solution
Let D(x) denote x is in this discrete mathematics class, and C(x) denote x takes a course in foundations of programming. The hypotheses are x.(D(x) C(x)) and D(Eva). The desired conclusion is C(Eva)
1

x.(D(x) C(x))

Premise

Valid arguments Examples

Fallacies

Rules of inference for quantied statements

Show that the premises Everyone in this discrete mathematics class also takes a course in foundations of programming, and Eva is a student is this class imply the conclusion Eva takes a course in foundations of programming.

Solution
Let D(x) denote x is in this discrete mathematics class, and C(x) denote x takes a course in foundations of programming. The hypotheses are x.(D(x) C(x)) and D(Eva). The desired conclusion is C(Eva)
1 2

x.(D(x) C(x)) D(Eva) C(Eva)

Premise Universal instantiation from (1)

Valid arguments Examples

Fallacies

Rules of inference for quantied statements

Show that the premises Everyone in this discrete mathematics class also takes a course in foundations of programming, and Eva is a student is this class imply the conclusion Eva takes a course in foundations of programming.

Solution
Let D(x) denote x is in this discrete mathematics class, and C(x) denote x takes a course in foundations of programming. The hypotheses are x.(D(x) C(x)) and D(Eva). The desired conclusion is C(Eva)
1 2 3

x.(D(x) C(x)) D(Eva) C(Eva) D(Eva)

Premise Universal instantiation from (1) Premise

Valid arguments Examples

Fallacies

Rules of inference for quantied statements

Show that the premises Everyone in this discrete mathematics class also takes a course in foundations of programming, and Eva is a student is this class imply the conclusion Eva takes a course in foundations of programming.

Solution
Let D(x) denote x is in this discrete mathematics class, and C(x) denote x takes a course in foundations of programming. The hypotheses are x.(D(x) C(x)) and D(Eva). The desired conclusion is C(Eva)
1 2 3 4

x.(D(x) C(x)) D(Eva) C(Eva) D(Eva) C(Eva)

Premise Universal instantiation from (1) Premise Modus ponens from (2) and (3)

Valid arguments Universal modus ponens & modus tollens

Fallacies

Rules of inference for quantied statements

Universal modus ponens & modus tollens

Universal modus ponens


x.(P(x) Q(x)) P(a), where a is a particular element in the domain Q(a)

Universal modus tollens


x.(P(x) Q(x)) Q(a), where a is a particular element in the domain P(a)

Valid arguments Universal modus ponens & modus tollens

Fallacies

Rules of inference for quantied statements

Example

Assume that For all positive integers n, if n is greater than 4, then n2 is less than 2n is true. Show that 1002 < 2100 .

Valid arguments Universal modus ponens & modus tollens

Fallacies

Rules of inference for quantied statements

Example

Assume that For all positive integers n, if n is greater than 4, then n2 is less than 2n is true. Show that 1002 < 2100 .

Solution
Let P(n) denote n > 4 and Q(n) denote n2 < 2n . The premise can be represented by n.(P(n) Q(n)), where the domain consists of all positive integers. Note P(100) is true because 100 > 4. It follows by universal modus ponens that Q(n) is true, namely 1002 < 2100

IKI10600I: Discrete Mathematics I


Proofs Adila A. Krisnadhi
Faculty of Computer Science, University of Indonesia

Basic Terminology of Mathematical Proof

Proof methods

Proofs
Outline

11

Basic Terminology of Mathematical Proof

12

Proof methods Direct proofs Proof by contraposition

Basic Terminology of Mathematical Proof

Proof methods

Another example (of previous lecture)


Is the following argument valid?
If Superman were able and willing to prevent evil, he would do so. If Superman were unable to prevent evil, he would be impotent; if he were unwilling to prevent evil, he would be malevolent. Superman does not prevent evil. If Superman exists, he is neither impotent nor malevolent. Therefore, Superman does not exist.

Basic Terminology of Mathematical Proof

Proof methods

Another example (of previous lecture)


Is the following argument valid?
If Superman were able and willing to prevent evil, he would do so. If Superman were unable to prevent evil, he would be impotent; if he were unwilling to prevent evil, he would be malevolent. Superman does not prevent evil. If Superman exists, he is neither impotent nor malevolent. Therefore, Superman does not exist.

Answer
Superman does not prevent evil. By Modus Tollens, we obtain that Superman is either unable or unwilling to prevent evil. If Superman is unable to prevent evil, then he is impotent; similarly, if Superman is unwilling to prevent evil, then he is malevolent. Thus, by Modus Ponens, we have that Superman is either impotent or malevolent. Since if Superman exists, he is neither impotent nor malevolent, by Modus Tollens, we conclude that Superman does not exist. Since the conclusion is derived by always using valid rules of inference, we know that the argument is valid.

Basic Terminology of Mathematical Proof

Proof methods

Basic Terminology I
Theorem, proposition, lemma: a statement that can be shown to be true. Theorem is usually reserved for a statement that is considered somewhat important; sometimes it is refered as fact or result. Propositions are typically less important than theorems; lemma are typically less important than propositions. Proof: a valid argument that establishes the truth of a theorem/proposition/lemma. Statements used in a proof can include:
axioms (postulates): statements assumed to be true; premise(s) of the theorem/proposition/lemma that we want to prove; previously proven theorem/proposition/lemma.

Basic Terminology of Mathematical Proof

Proof methods

Basic Terminology II

Lemma (small theorem) are usually used to help proving other results (not as itself); complicated proofs are usually easier to understand when they are proved using series of lemmas. Corollary: a theorem that can be established directly/straightforwardly from a theorem that has been proved. Conjencture: a statement that is being proposed to be a true statement; its truth is unknown until somebody gives it a valid proof.

Basic Terminology of Mathematical Proof

Proof methods

Theorem statement
Many (though not all) theorems assert that a property holds for all elements in a domain. In such cases, a universal quantier is usually omitted, although it is formally needed for a precise statement. In proving theorems, universal instantiation is often used implicitly.

Basic Terminology of Mathematical Proof

Proof methods

Theorem statement
Many (though not all) theorems assert that a property holds for all elements in a domain. In such cases, a universal quantier is usually omitted, although it is formally needed for a precise statement. In proving theorems, universal instantiation is often used implicitly. The theorem: If x > y , where x and y are positive real numbers, then x 2 > y 2

Basic Terminology of Mathematical Proof

Proof methods

Theorem statement
Many (though not all) theorems assert that a property holds for all elements in a domain. In such cases, a universal quantier is usually omitted, although it is formally needed for a precise statement. In proving theorems, universal instantiation is often used implicitly. The theorem: If x > y , where x and y are positive real numbers, then x 2 > y 2 actually means

Basic Terminology of Mathematical Proof

Proof methods

Theorem statement
Many (though not all) theorems assert that a property holds for all elements in a domain. In such cases, a universal quantier is usually omitted, although it is formally needed for a precise statement. In proving theorems, universal instantiation is often used implicitly. The theorem: If x > y , where x and y are positive real numbers, then x 2 > y 2 actually means For all positive real numbers x and y , if x > y , then x 2 > y 2

Basic Terminology of Mathematical Proof Direct proofs

Proof methods

Direct proof

A direct proof of a statement p q is constructed as follows:

Basic Terminology of Mathematical Proof Direct proofs

Proof methods

Direct proof

A direct proof of a statement p q is constructed as follows:


rst, assume that p is true;

Basic Terminology of Mathematical Proof Direct proofs

Proof methods

Direct proof

A direct proof of a statement p q is constructed as follows:


rst, assume that p is true; construct subsequent steps using rules of inference until ...

Basic Terminology of Mathematical Proof Direct proofs

Proof methods

Direct proof

A direct proof of a statement p q is constructed as follows:


rst, assume that p is true; construct subsequent steps using rules of inference until ... q is obtained as true as the nal step.

Basic Terminology of Mathematical Proof Direct proofs

Proof methods

Direct proof

A direct proof of a statement p q is constructed as follows:


rst, assume that p is true; construct subsequent steps using rules of inference until ... q is obtained as true as the nal step.

In intermediate steps, we are also allowed to use axioms, denitions and previously proven theorems.

Basic Terminology of Mathematical Proof Direct proofs

Proof methods

Direct proof

A direct proof of a statement p q is constructed as follows:


rst, assume that p is true; construct subsequent steps using rules of inference until ... q is obtained as true as the nal step.

In intermediate steps, we are also allowed to use axioms, denitions and previously proven theorems. A direct proof of a statement x(P(x) Q(x)) is constructed by showing that P(c) Q(c) is true, where c is an arbitrary element of the domain (and then applying universal generalization).

Basic Terminology of Mathematical Proof Direct proofs

Proof methods

Prove the following theorems using direct proof


see the whiteboard for details; take notes if necessary!

Denition
An integer n is even if there exists an integer k such that n = 2k ; and n is odd if there exists an integer k such that n = 2k + 1.

Theorem
If n is an odd integer then n2 is odd.

Basic Terminology of Mathematical Proof Direct proofs

Proof methods

Prove the following theorems using direct proof


see the whiteboard for details; take notes if necessary!

Theorem
If m and n are both perfect squares, then then nm is also perfect square, where an integer a is a perfect square if there is an integer b such that a = b2 .

Theorem
The sum of two odd integers is even. The square of an even number is an even number.

Theorem
If m + n and n + p are even integers, where m, n, and p are integers, then m + p is even.

Basic Terminology of Mathematical Proof Direct proofs

Proof methods

Prove the following theorems using direct proof


see the whiteboard for details; take notes if necessary!

Denition
A real number r is rational if there exists integers p and q with p q = 0 such that r = q ; otherwise it is called irrational.

Theorem
The sum of two rational numbers is rational.

Basic Terminology of Mathematical Proof Proof by contraposition

Proof methods

Proof by contraposition

A proof by contraposition for a statement p q is constructed as follows:

Basic Terminology of Mathematical Proof Proof by contraposition

Proof methods

Proof by contraposition

A proof by contraposition for a statement p q is constructed as follows:


assume that q is true;

Basic Terminology of Mathematical Proof Proof by contraposition

Proof methods

Proof by contraposition

A proof by contraposition for a statement p q is constructed as follows:


assume that q is true; construct subsequent steps using rules of inference until ...

Basic Terminology of Mathematical Proof Proof by contraposition

Proof methods

Proof by contraposition

A proof by contraposition for a statement p q is constructed as follows:


assume that q is true; construct subsequent steps using rules of inference until ... p is obtained as true as the nal step.

Basic Terminology of Mathematical Proof Proof by contraposition

Proof methods

Prove the following theorems by contraposition


see the whiteboard for details; take notes if necessary!

Theorem
If n is an integer and 3n + 2 is odd, then n is odd.

Theorem
If n = ab, where a and b are positive integers, then a b n.

n or

Theorem
If n is an integer and n2 is odd, then n is odd.

Theorem
If x is irrational, then 1/x is irrational.

IKI10600I: Discrete Mathematics I


Proofs (2)

Adila A. Krisnadhi
Faculty of Computer Science, University of Indonesia

Proof Methods (2)

Mistakes in Proof

Proof methods (3)

Proofs (2)
Outline

13

Proof Methods (2) Vacuous and Trivial Proofs Proof by Contradiction Proofs of equivalence Counterexamples Mistakes in Proof Proof methods (3) Exhaustive proof Proof by cases

14 15

Proof Methods (2) Vacuous and Trivial Proofs

Mistakes in Proof

Proof methods (3)

Vacuous proof

We can quickly prove that a conditional statement p q is true when we know that p is false. Why?

Proof Methods (2) Vacuous and Trivial Proofs

Mistakes in Proof

Proof methods (3)

Vacuous proof

We can quickly prove that a conditional statement p q is true when we know that p is false. Why? Because p q must be true whenever p is false.

Proof Methods (2) Vacuous and Trivial Proofs

Mistakes in Proof

Proof methods (3)

Vacuous proof

We can quickly prove that a conditional statement p q is true when we know that p is false. Why? Because p q must be true whenever p is false. Show that the proposition P(0) is true, where P(n) is If n > 1, then n2 > n and the domain consists of all integers.

Proof Methods (2) Vacuous and Trivial Proofs

Mistakes in Proof

Proof methods (3)

Vacuous proof

We can quickly prove that a conditional statement p q is true when we know that p is false. Why? Because p q must be true whenever p is false. Show that the proposition P(0) is true, where P(n) is If n > 1, then n2 > n and the domain consists of all integers.

Vacuous proof
Note that P(0) is If 0 > 1, then 02 > 0. Then P(0) is true because the hypothesis 0 > 1 is false.

Proof Methods (2) Vacuous and Trivial Proofs

Mistakes in Proof

Proof methods (3)

Trivial proof
We can quickly prove that a conditional statement p q is true when we know that q is true. Why?

Proof Methods (2) Vacuous and Trivial Proofs

Mistakes in Proof

Proof methods (3)

Trivial proof
We can quickly prove that a conditional statement p q is true when we know that q is true. Why? Because p q must be true whenever q is true (regardless of the hypothesis p).

Proof Methods (2) Vacuous and Trivial Proofs

Mistakes in Proof

Proof methods (3)

Trivial proof
We can quickly prove that a conditional statement p q is true when we know that q is true. Why? Because p q must be true whenever q is true (regardless of the hypothesis p). Let P(n) be If a and b are positive integers with a b, then an bn where the domain consists of all integers. Show that P(0) is true.

Proof Methods (2) Vacuous and Trivial Proofs

Mistakes in Proof

Proof methods (3)

Trivial proof
We can quickly prove that a conditional statement p q is true when we know that q is true. Why? Because p q must be true whenever q is true (regardless of the hypothesis p). Let P(n) be If a and b are positive integers with a b, then an bn where the domain consists of all integers. Show that P(0) is true.

Trivial proof
Note that P(0) is If a b, then a0 b0 . Because a0 = b0 = 1, the conclusion of the statement P(0) is true. Hence, P(0) itself is trivially true. Note that the hypothesis a b was not needed in this proof.

Proof Methods (2) Proof by Contradiction

Mistakes in Proof

Proof methods (3)

Proof by contradiction: idea


The idea of proof by contradiction: Suppose we want to prove that a statement p is true.

Proof Methods (2) Proof by Contradiction

Mistakes in Proof

Proof methods (3)

Proof by contradiction: idea


The idea of proof by contradiction: Suppose we want to prove that a statement p is true. Moreover, suppose we can show that there is a statement q such that p q is true AND q is a contradictory statement.

Proof Methods (2) Proof by Contradiction

Mistakes in Proof

Proof methods (3)

Proof by contradiction: idea


The idea of proof by contradiction: Suppose we want to prove that a statement p is true. Moreover, suppose we can show that there is a statement q such that p q is true AND q is a contradictory statement. Then, we can conclude that p is false, which means that p is true.

Proof Methods (2) Proof by Contradiction

Mistakes in Proof

Proof methods (3)

Proof by contradiction: idea


The idea of proof by contradiction: Suppose we want to prove that a statement p is true. Moreover, suppose we can show that there is a statement q such that p q is true AND q is a contradictory statement. Then, we can conclude that p is false, which means that p is true. How can we nd such a contradiction q that might help us prove that p is true in this way?

Proof Methods (2) Proof by Contradiction

Mistakes in Proof

Proof methods (3)

Proof by contradiction: idea


The idea of proof by contradiction: Suppose we want to prove that a statement p is true. Moreover, suppose we can show that there is a statement q such that p q is true AND q is a contradictory statement. Then, we can conclude that p is false, which means that p is true. How can we nd such a contradiction q that might help us prove that p is true in this way? Because the statement r r is always a contradiction whenever r is a proposition, we can prove that p is true if we can show that p (r r ) is true for some proposition r .

Proof Methods (2) Proof by Contradiction

Mistakes in Proof

Proof methods (3)

Proof by contradiction: steps

Suppose that p is a proposition that we want to prove. A proof by contradiction of p is constructed as follows: First, assume that p is true.

Proof Methods (2) Proof by Contradiction

Mistakes in Proof

Proof methods (3)

Proof by contradiction: steps

Suppose that p is a proposition that we want to prove. A proof by contradiction of p is constructed as follows: First, assume that p is true. Derive statements which are true as consequences of this assumption until ...

Proof Methods (2) Proof by Contradiction

Mistakes in Proof

Proof methods (3)

Proof by contradiction: steps

Suppose that p is a proposition that we want to prove. A proof by contradiction of p is constructed as follows: First, assume that p is true. Derive statements which are true as consequences of this assumption until ... we obtain a statement that contradicts previously-derived statements, i.e., a statement r whose negation r is already derived previously.

Proof Methods (2) Proof by Contradiction

Mistakes in Proof

Proof methods (3)

Prove the following theorems by contradiction


see the whiteboard for details; take notes if necessary!

Theorem
At least four of any 22 days must fall on the same day.

Theorem
2 is irrational

Theorem
If 3n + 2 is odd, then n is odd.

Proof Methods (2) Proofs of equivalence

Mistakes in Proof

Proof methods (3)

Proof of equivalence

In order to show that a biconditional statement p q is true, we show that p q and q p are both true.

Proof Methods (2) Proofs of equivalence

Mistakes in Proof

Proof methods (3)

Proof of equivalence

In order to show that a biconditional statement p q is true, we show that p q and q p are both true. This approach is valid because it is based on the tautology (p q) [(p q) (q p)].

Proof Methods (2) Proofs of equivalence

Mistakes in Proof

Proof methods (3)

Prove the following theorems


see the whiteboard for details; take notes if necessary!

Theorem
If n is a positive integer, then n is odd iff n2 is odd.

Theorem
If n is a positive integer, then n is odd if and only if 5n + 6 is odd.

Proof Methods (2) Proofs of equivalence

Mistakes in Proof

Proof methods (3)

Proof of equivalence (more than 2 statements)


In order to show that the propositions p1 , p2 , . . . , pn are equivalent, we show that p1 p2 . . . pn is true. One way to show this, is by showing that all conditional statements p1 p2 , p2 p3 , . . . , pn p1 are true. The idea is so that we can establish any chain of conditional statements from any one of the proposition to any other propositions.

Show the following statements are equivalent


1 2 3

n is even. n 1 is odd. n2 is even.

Proof Methods (2) Counterexamples

Mistakes in Proof

Proof methods (3)

Counterexamples
In order to show that a statement of the form xP(x), we need only nd a counterexample, i.e., an example x for which P(x) is false.

Using counterexample
Disprove the statement Every positive integer is the sum of the squares of two integers.

Proof Methods (2) Counterexamples

Mistakes in Proof

Proof methods (3)

Counterexamples
In order to show that a statement of the form xP(x), we need only nd a counterexample, i.e., an example x for which P(x) is false.

Using counterexample
Disprove the statement Every positive integer is the sum of the squares of two integers.

Answer
To disprove the universally quantied statement above, we look for a counterexample which is a particular integer that is NOT the sum of the squares of two integers. For this statement, a counterexample is easy to nd. One such counterexample is 3. To show that this is the case, not that the only two perfect squares not exceeding 3 are 02 = 0 and 12 = 1. It is easy to see that there is no way to get 3 as the sum of two terms each of which is 0 or 1. Thus 3 is a counterexample of the statement above.

Proof Methods (2)

Mistakes in Proof

Proof methods (3)

Fallacy: arming the conclusion


Can you nd what is wrong in the following proof of the statement If n2 is positive, then n is positive?

Supposed Proof
Suppose that n2 is positive. Because the statement If n is positive, then n2 is positive is true, we can conclude that n is positive.

Proof Methods (2)

Mistakes in Proof

Proof methods (3)

Fallacy: arming the conclusion


Can you nd what is wrong in the following proof of the statement If n2 is positive, then n is positive?

Supposed Proof
Suppose that n2 is positive. Because the statement If n is positive, then n2 is positive is true, we can conclude that n is positive. This proof is wrong. Let P(n) be n is positive and Q(n) be n2 is positive. Then the statement that we want to prove is n(Q(n) P(n)). Take Q(n) is the hypothesis. In the supposed proof, we make use of the statement n(P(n) Q(n)). But from Q(n) as the hypothesis and the statement n(P(n) Q(n)), we cannot conclude P(n), since there is no valid rule inference that is usable here.

Proof Methods (2)

Mistakes in Proof

Proof methods (3)

Fallacy: begging the question (circular reasoning)


Can you nd what is wrong with the following proof of the theorem n2 is an even integer implies n is an even integer?

Supposed proof
Suppose that n2 is even. Then n2 = 2k for some integer k . Let n = 2 for some integer . This shows that n is even.

Proof Methods (2)

Mistakes in Proof

Proof methods (3)

Fallacy: begging the question (circular reasoning)


Can you nd what is wrong with the following proof of the theorem n2 is an even integer implies n is an even integer?

Supposed proof
Suppose that n2 is even. Then n2 = 2k for some integer k . Let n = 2 for some integer . This shows that n is even. The proof is wrong, because the statement let n = 2 for some integer occurs in the proof. No argument has been given to show that n can indeed be written as 2 for some integer . This is circular reasoning because this statement is equivalent to the statement being proved, i.e., n is even. Note that the theorem itself is actually correct (can you give a valid proof of it?), but the method of proof is wrong.

Proof Methods (2) Exhaustive proof

Mistakes in Proof

Proof methods (3)

Some statement can be proved by examining ALL possible example. Such proofs is called exhaustive proof. This method is suitable only when the number of examples are relatively small.

Prove the following result


The only consecutive positive integers not exceeding 100 that are perfect powers are 8 and 9. (An integer n is a perfect power if there exists two integers m and k > 1 such that n = mk ).

Proof Methods (2) Exhaustive proof

Mistakes in Proof

Proof methods (3)

Some statement can be proved by examining ALL possible example. Such proofs is called exhaustive proof. This method is suitable only when the number of examples are relatively small.

Prove the following result


The only consecutive positive integers not exceeding 100 that are perfect powers are 8 and 9. (An integer n is a perfect power if there exists two integers m and k > 1 such that n = mk ). An exhaustive proof of this statement can be done by constructing a table consisting all possible example of numbers not exceeding 100 and of the form mk . This method is suitable only because the size of the table is relatively small. If you must prove the result for integers not exceeding 1000, this method would be too difcult and tedious.

Proof Methods (2) Proof by cases

Mistakes in Proof

Proof methods (3)

Proof by cases
see the whiteboard for details; take notes if necessary!

A proof by cases must cover ALL possible cases that arise in a theorem. This method works since the statement (p1 p2 . . . pn ) q is equivalent to (p1 q) (p2 q) . . . (pn q).

Theorem
If n is an integer, then n2 n.

Theorem
For every real numbers x and y , |xy | = |x||y |, where |a| = a if a 0 and |a| = a if a < 0.

IKI10600I: Discrete Mathematics I


Set Adila A. Krisnadhi
Faculty of Computer Science, University of Indonesia

Set: basics

Set operations

Set
Outline

16

Set: basics

17

Set operations

Set: basics

Set operations

Set: denition

Denition
A set is an unordered collection of distinct objects.

Denition
Objects in a set are called elements or members of the set. Here, the set is said to contain its elements.

Set: basics

Set operations

Set: notation

Sets are usually denoted by large capitals: A, B, X , Y , . . . ; if necessary with index. Elements of a set are usually denoted by small letters: a, b, x, y , . . . ; if necessary with index. x A denotes that x is an element of A, i.e., A contains x as one of its element. x A denotes that x is not an element of A. / denotes the empty set.

Set: basics

Set operations

Set: representation

Representation of sets: using listing:


{x1 , x2 , x3 . . . , xn } for sets with nite number of elements {x1 , x2 , x3 . . . } for sets with innite number of elements

using description (set builder notation) where P is a predicate over elements of the set:
{x : P(x)} or {x | P(x)} {x S : P(x)} or {x S | P(x)} where S is another set in the context of the discussion, restricting the elements of the denoted set (sometimes called a universal set).

Set: basics

Set operations

Set: examples
The set of natural numbers N = {0, 1, 2, 3, . . . } (Note: sometimes people dene natural numbers starting from 1, not 0) The set of integers Z = {0, 1, 1, 2, 2, . . . } = {. . . , 2, 1, 0, 1, 2, . . . } The set of positive integers Z+ = {1, 2, 3, . . . } The set of rational numbers Q = {p/q | p Z, q Z, and q = 0} The set of real numbers R A = {x N | x 2 3x + 2 = 0}, set of natural numbers x such that x 2 3x + 2 = 0. A can also be written as {1, 2}. The set of positive even numbers is written as {2x | x N} or {x | x = 2y , y N} or {x N | x = 2y , y N}. The set B = {x N | 6 < x < 9} can also be written as B = {7, 8} The set B = {7, 8} can also be written as {x N | 40 < x 2 < 80} or {x N | x 2 15x + 56 = 0} or {7 + x N | x = 0 or x = 1}

Set: basics

Set operations

Set equality and subsets


Denition
Two sets A and B are equal (written A = B) iff A and B have the same elements. Otherwise, they are not equal and written as A = B. A = B iff x(x A x B) is true.

Denition
The set A is a subset of B (written A B) iff every element of A is also an element of B. Here, we also say that B is a superset of A (written B A) A B iff x(x A x B) is true.

Denition
The set A is a proper subset of B iff A B and A = B

Set: basics

Set operations

Set equality and subsets (contd.)


{1, 3, 5} = {3, 5, 1} (the order is not important) {1, 3, 5} {3, 5, 1} {1, 3, 5} N {1, 3, 5} {3, 1} and {1, 3, 5} {x N | x is even}

Theorem (proof is for exercise!)


For every set S S SS

Theorem (proof is for exercise!)


For every two sets A and B, A = B iff A B and B A

Set: basics

Set operations

Finite set and cardinality


Denition
Let S be a set. S is a nite set iff it has exactly n elements where n is nonnegative integer. Here, n is called the cardinality of S. The cardinality of S is denoted by |S| S is innite iff it is not nite. Let A be the set of odd positive integer less than 10. Then A is nite (its cardinality |A| is 5). || = 0. The set of positive integers is innite.

Set: basics

Set operations

The power set


Denition
Let S be a set. The power set of S is the set of all subsets of S. The power set of S is denoted by P(S) or sometimes 2S . The power set of the set {0, 1, 2} is P({0, 1, 2}) = {, {0}, {1}, {2}, {0, 1}, {0, 2}, {1, 2}, {0, 1, 2}} The power set of the empty set is P() = {}. The power set of the set {} is P({}) = {, {}}

Theorem
If a set S has n elements then its power set P(S) has 2n elements.

Set: basics

Set operations

Ordered tuple
Sets are unordered; we need different structure to represent ordered collection.

Denition
The (ordered) n-tuple (a1 , a2 , . . . , an ) is the ordered collection with a1 as its rst element, a2 as its second element, ..., and an as its nth element. 2-tuples are usually called ordered pairs Two n-tuples (a1 , a2 , . . . , an ) and (b1 , b2 , . . . , bn ) are equal iff ai = bi for i = 1, 2, . . . , n (i.e., each corresponding pair of their elements are equal). In particular, two ordered pairs (a, b) and (c, d) are equal iff a = c and b = d.

Set: basics

Set operations

Cartesian products
We can use Cartesian product to form a set of ordered pairs from two sets.

Denition
Let A and B be sets. The Cartesian product of A and B (denoted by A B), is the set of all ordered pairs (a, b) where a A and b B. In other words A B = {(a, b) | a A b B}
If A = {1, 2, 3} and B = {2, 3}, then A B = {(1, 2), (1, 3), (2, 2), (2, 3), (3, 2), (3, 3)} If A = {x | 2 x 3} and B = {x | 1 x 2 or 3 x 4}, then A B = {(x, y ) | 2 x 3 and 1 y 2 or 3 y 4} A B = B A unless A = B or A = or B =

Set: basics

Set operations

Relation

Denition
A relation R from a set A to a set B is a set of ordered pairs (a, b) such that a is some element of a and b is some element B. In other words, R A B. Let A = {1, 2, 3} and B = {2, 3}. R = {(1, 3), (3, 3)} is a relation from A to B. Let A = {1, 2, 3} and B = {a, b, c}. R = {(1, a), (1, b), (1, c)} is a relation from A to B.

Set: basics

Set operations

Cartesian products of more than two sets

Denition
The Cartesian product of the sets A1 , A2 , . . . , An , (denoted by A1 A2 An ) is the set of ordered n-tuples (a1 , a2 , . . . , an ), where ai Ai for i = 1, 2, . . . , n. In other words, A1 A2 An = {(a1 , a2 , . . . , an ) | ai Ai for i = 1, 2, . . . , n}
Let A = {0, 1}, B = {1, 2} and C = {0, 1, 2}. A B C = {(0, 1, 0), (0, 1, 1), (0, 1, 2), (0, 2, 0), (0, 2, 1), (0, 2, 2), (1, 1, 0), (1, 1, 1), (1, 1, 2), (1, 2, 0), (1, 2, 1), (1, 2, 2)}

Set: basics

Set operations

Venn diagram

Let V = {a, e, i, o, u} be the set of all vowels. A Venn diagram for V is:

Set: basics

Set operations

Set union, intersection, difference and complement


Denition
The union of the sets A and B is A B = {x | x A x B}. The intersection of the sets A and B is A B = {x | x A x B}. Two sets A and B are disjoint iff A B = . The difference of the sets A and B is A B = A \ B = {x | x A x B}. / A \ B is also called the complement of B with respect to A. Assuming that we have a universal set S, the complement of a set A is AC = A = S \ A = {x | x A}. /

Set: basics

Set operations

Venn diagrams of set operations

Set: basics

Set operations

Set operations: examples

Let A = {1, 3, 5} and B = {1, 2, 4} A B = {1, 3, 5, 2, 4} A B = {1} A \ B = {3, 5} B \ A = {2, 4} A and B are NOT disjoint. But A and {2, 4} are disjoint. If the universal set U = {x N | 1 x 10}, then A = {2, 4, 6, 7, 8, 9, 10}

Set: basics

Set operations

Set identities
Proofs of them are left as exercises

Identity laws: A = A and A U = A for a universal set U Domination laws: A U = U for a universal set U and A = Idempotent laws: A A = A and A A = A. Complementation law: A = A. Complement laws: A A = U for a universal set U, and A A = Commutative laws: A B = B A and A B = B A. Associative laws: A (B C) = (A B) C and A (B C) = (A B) C. Distributive laws: A (B C) = (A B) (A C) and A (B C) = (A B) (A C). De Morgans laws: A B = A B and A B = A B Absorption laws: A (A B) = A and A (A B) = A.

Set: basics

Set operations

Generalized unions and intersections


Due to associativity, A (B C) or (A B) C can be written as A B C. Similarly, A (B C) and (A B) C can be written as A B C. For n 3 sets A1 , A2 , . . . , An , their intersection can be written as
n

Ai =
i=1

{Ai , i = 1, 2, . . . , n} = A1 A2 . . . An

and their union can be written as


n

Ai =
i=1

{Ai , i = 1, 2, . . . , n} = A1 A2 . . . An

IKI10600I: Discrete Mathematics I


Functions, Sequences and Summations

Adila A. Krisnadhi
Faculty of Computer Science, University of Indonesia

Functions

Sequences and Summations

Cardinality of set: revisited

Functions, Sequences and Summations


Outline

18

Functions Sequences and Summations Sequences Summations Cardinality of set: revisited

19

20

Functions

Sequences and Summations

Cardinality of set: revisited

Function: denitions

Denition
Let A and B be nonempty sets. A function f from A to B (written f : A B) is an assignment of each element of A to exactly one element of B. Functions are also called mappings or transformations We write f (a) = b if a A is assigned to a unique b B A function can also be viewed as a relation from A to B that contains one and only one ordered pair (a, b) for evey element a A.

Functions

Sequences and Summations

Cardinality of set: revisited

Domain, codomain and range


If f is a function from A to B, we say that
A is the domain of f B is the codomain of f

If f (a) = b, we say that


b is the image of a a is the preimage of b

The range of f is the set of all images of elements of A. The range of f is always a subset of the codomain of f If f is a function from A to B, we say that f maps A to B. Two functions are equal if they have the same domain, the same codomain and map elements of their common domain to the same elements in their common codomain. Equality of functions can also be seen as equality of sets (recall the denition of function as relation).

Functions

Sequences and Summations

Cardinality of set: revisited

More basics on functions


Let f1 and f2 be functions from A to R. Then
f1 + f2 is a function from A to R dened by (f1 + f2 )(x) = f1 (x) + f2 (x) f1 f2 is a function from A to R dened by (f1 f2 )(x) = f1 (x)f2 (x)

Let f be a function from A to B and S A. The image of S under f is the subset of B that consists of the images of elements of S, i.e., {t | s S(t = f (s))} Note: we can also use a shorthand for this set: {f (s) | s S}

Functions

Sequences and Summations

Cardinality of set: revisited

Increasing and decreasing functions


Let f : A B be a function. f is increasing iff x, y A : x < y f (x) f (y ) f is strictly increasing iff x, y A : x < y f (x) < f (y ) f is decreasing iff x, y A : x < y f (x) f (y ) f is strictly decreasing iff x, y A : x < y f (x) > f (y )

Lemma
Let f : A B be a function. If f is strictly increasing, then f is increasing. If f is strictly decreasing, then f is decreasing.

Functions

Sequences and Summations

Cardinality of set: revisited

Surjective, injective and bijective functions

Let f : A B be a function. f is surjective (onto) iff it satises y B.x A : f (x) = y f is injective (one-to-one) iff it satises x1 x2 (f (x1 ) = f (x2 ) x1 = x2 ) f is bijective (one-to-one correspondence) iff f is both surjective and injective.

Theorem
Let f : A B be a function. If f is either strictly increasing or strictly decreasing, then f is a one-to-one function.

Functions

Sequences and Summations

Cardinality of set: revisited

Examples
Let f be a function from {a, b, c, d} to {1, 2, 3, 4, 5} with f (a) = 4, f (b) = 5, f (c) = 1 and f (d) = 3. Then f is one-to-one, but not onto. Let f : Z Z be a function such that f (x) = x 2 . Then
f is not one-to-one, because e.g., f (1) = f (1) = 1, but 1 = 1; f is not onto, because there is no x Z such that f (x) = 1 f is not increasing and not decreasing. Why?

Let f : Z Z be a function such that f (x) = x + 1. Then


f is one-to-one, because for every x1 , x2 Z, x1 + 1 = x2 + 1 implies x1 = x2 . f is onto, because for every y Z, there is an x Z such that f (x) = y by taking x = y 1. f is bijective. Why? f is strictly increasing. Why?

Functions

Sequences and Summations

Cardinality of set: revisited

Inverse function
Denition
Let f : A B be a bijective function. The inverse function of f is the function f 1 : B A dened such that if f (x) = y then f 1 (y ) = x. Do not confuse f 1 with the function 1/f . They are NOT the same. Inverse function of f only exists (is well-dened) if f is bijective. Why? If f is bijective, then f has an inverse function, i.e., f is invertible.

Functions

Sequences and Summations

Cardinality of set: revisited

Examples
Let f be the function from {a, b, c} to {1, 2, 3} such that f (a) = 2, f (b) = 3 and f (c) = 1. Then f is invertible (why?) and the inverse of f is the function f 1 from {1, 2, 3} to {a, b, c} such that f 1 (1) = c, f 1 (2) = a, and f 1 (3) = b. Let f : Z Z be such that f (x) = x + 1. Then f is invertible (why?) and its inverse is the function f 1 : Z Z such that f 1 (x) = x 1. Let f : R R be such that f (x) = x 2 . Is f invertible? If it is, what is its inverse? Let f : R+ {0} R+ {0} be such that f (x) = x 2 . Is f invertible? If it is, what is its inverse?

Functions

Sequences and Summations

Cardinality of set: revisited

Function composition

Denition
Let f : B C and g : A B be functions. The composition of f and g is the function f g : A C dened as (f g)(x) = f (g(x)) To nd (f g)(a), we rst apply g to a obtaining g(a) and then we apply f to g(a) to obtain (f g)(a). For the function f g to be well-dened, the range of g must be a subset of the domain of f . Why?

Functions

Sequences and Summations

Cardinality of set: revisited

Examples

Let g be the function from {a, b, c} to itself such that g(a) = b, g(b) = c and g(c) = a. Let f be the function from {a, b, c} to {1, 2, 3} such that f (a) = 3, f (b) = 2 and f (c) = 1. Then
(f g) is the function from {a, b, c} to {1, 2, 3} such that (f g)(a) = f (g(a)) = 2, (f g)(b) = f (g(b)) = 1 and (f g)(c) = f (g(c)) = 3 Is g f well-dened? If it is, give the denition of g f .

Let f , g : Z Z be functions such that f (x) = 2x + 3 and g(x) = 3x + 2. What is f g? What is g f ?

Functions

Sequences and Summations

Cardinality of set: revisited

Some theorems on composition and inverse

f g does not equal g f in general. For a function f : A B, if f 1 exists, then


f f 1 = idA where id : A A is the identity function on A, i.e., id(x) = x; f 1 f = idB where id : B B is the identity function on A, i.e., id(x) = x; If A = B, then f f 1 = f 1 f = idA (f 1 )1 = f .

Functions Sequences

Sequences and Summations

Cardinality of set: revisited

Sequences
Denition (Informal denition)
A sequence is an ordered list of terms.

Denition (Formal denition)


A sequence is a function from a subset of the set of integers (typically the set {0, 1, 2, . . . } or {1, 2, 3, . . . }) to a set S. We use the notation {an } to denote a sequence (DO NOT confuse it with sets) where we can either describe the form of an and/or list the terms of the sequence in order of increasing subscripts.

Functions Sequences

Sequences and Summations

Cardinality of set: revisited

Examples
The sequence {an } where an = 1/n for n = 1, 2, . . . is the sequence 1, 1 , 1 , 1 , . . . . 2 3 4 A geometric progression is the sequence of the form a, ar , ar 2 , . . . , ar n , . . . with the initial term a and the ratio r are real numbers. For example:
1, 2, 4, 8, . . . . 1 1, 2 , 1 , 1 , . . . . 4 8 1, 1, 1, 1, 1, . . . .

An arithmetic progression is the sequence of the form a, a + d, a + 2d, . . . , a + nd, . . . where the initial term a and the common difference r are real numbers. For example:
1, 3, 7, 11, . . . . 7, 4, 1, 2, . . . .

Functions Sequences

Sequences and Summations

Cardinality of set: revisited

Deducing formula for terms of a sequence


When you want to deduce a formula for the terms of a sequence, try to answer the following questions: Are there runs of the same value? I.e., does the same value occur many times in a row? Are terms obtained from previous terms by adding the same amount or an amount that depends on the position in the sequence? Are terms obtained from previous terms by multiplying by a particular amount? Are terms obtained by combining previous terms in a certain way? Are there cycles among the terms?

Functions Sequences

Sequences and Summations

Cardinality of set: revisited

Examples

Find the formulae for the sequences:


1, 1/2, 1/4, 1/8, . . . . 1, 3, 5, 7, 9, . . . . 1, 1, 1, 1, . . . .

Continue the following sequence with appropriate terms:


1, 2, 2, 3, 3, 3, 4, 4, 4, 4, . . . . 5, 11, 17, 23, 29, 35, 41, 47, 53, 59, . . . .

Guess a simple formula for an if the rst 10 terms of the sequence {an } are 1, 7, 25, 79, 241, 727, 2185, 6559, 19681, 59047.

Functions Summations

Sequences and Summations

Cardinality of set: revisited

Summation
We are usually interested in summing a given sequence. Here, we use the summation notation, namely the sigma notation. Given the terms am , am+1 , . . . , an from the sequence {an }, we use the notation
n

aj ,
j=m

n j=m

aj ,

or

n mjn

aj

to represent am + am+1 + + an In the notation, j is the index of summation, and we could also use any other letter such as i or k . Also, m is the lower limit and n is the upper limit of the summation.

Functions Summations

Sequences and Summations

Cardinality of set: revisited

Example
The sum of the rst 100 terms of the sequences {an } where an = 1/n for n = 1, 2, 3, . . . is 100 1 . j=1 j Calculate Calculate
5 2 j=1 j . 8 k k =4 (1) .

Index of summation can be shifted, e.g., 5 4 2 2 j=1 j = k =0 (k + 1) Prove this result: if a and r ar real numbers, and r = 0, then n ar n+1 a if r = 1 r 1 ar j = (n + 1)a if r = 1 j=0

Functions Summations

Sequences and Summations

Cardinality of set: revisited

Nested summations and summations over set


Summations can be nested. E.g.:
4 3 4

ij =
i=1 j=1 i=1 4

(i + 2i + 3i)

=
i=1

6i

= 6 + 12 + 18 + 24 = 60

Functions Summations

Sequences and Summations

Cardinality of set: revisited

Nested summations and summations over set


Summations can be nested. E.g.:
4 3 4

ij =
i=1 j=1 i=1 4

(i + 2i + 3i)

=
i=1

6i

= 6 + 12 + 18 + 24 = 60 Given a set S, we can sum the values f (s) for all members of S. E.g.: s2 = 02 + 22 + 42 = 20
s{0,2,4}

Functions Summations

Sequences and Summations

Cardinality of set: revisited

Some useful summations


n

Provided r = 0:
k =0

ar k =

ar n+1 a r 1

(n + 1)a

if r = 1 if r = 1

(a + kd) = (n + 1)a +
k =0 n

dn(n + 1) 2

k=
k =1 n

n(n + 1) 2 n(n + 1)(2n + 1) 6 n2 (n + 1)2 4

k2 =
k =1 n

k3 =
k =1

Functions

Sequences and Summations

Cardinality of set: revisited

Cardinality of set: revisited

Recall that, cardinality of a set is dened as the number of elements in the set. Two nite sets A and B are of the same cardinality if the number of elements in A is the same as the number of elements in B (which can be counted). What if the set is innite? Can we extend the notion of cardinality to innite sets?

Functions

Sequences and Summations

Cardinality of set: revisited

Cardinality of a set: alternative denition

Denition
The sets A and B have the same cardinality iff there exists a bijection from A to B. A set is countable iff it is either nite or has the same cardinality with the set of positive integers (Z+ ). A set is uncountable iff it is not countable. When an innite set S is countable, we denote the cardinality of S (i.e., |S|) by 0 (read as aleph null)

Functions

Sequences and Summations

Cardinality of set: revisited

Examples

Is the set of odd positive integers is countable? Explain. Is the set {5, 4, . . . , 9, 10} countable? Explain. Is the set of all integers (Z) is countable? Explain. Is the set of positive rational numbers is countable? Explain. Is the set of real numbers is countable? Explain.

Functions

Sequences and Summations

Cardinality of set: revisited

More examples

Is a subset of a countable set also countable? Suppose A and B are sets, A is uncountable, and A B. Is B countable or uncountable? Prove it. Suppose A is an uncountable set and B is a countable set. Is A \ B countable or uncountable? Prove it.

IKI10600I: Discrete Mathematics I


Integers

Adila A. Krisnadhi
Faculty of Computer Science, University of Indonesia

The Integers Division

Primes and Greatest Common Divisor

Integers
Outline

21

The Integers Division Division The Division Algorithm Modular Arithmetic Primes and Greatest Common Divisor Primes Greatest Common Divisor and Least Common Multiples

22

The Integers Division Division

Primes and Greatest Common Divisor

Division

From your previous experiences, you should already know some properties of integers: on addition, multiplication, etc. We are going to look at an operation on integers in more details, namely the division. When an integer is divided by another, nonzero integer, the result (i.e., quotient) may or may not be an integer. E.g., 12/4 = 3, whereas 11/4 = 2.75. So, if we require the division operation on integers always to give an integer, we need the following denition

The Integers Division Division

Primes and Greatest Common Divisor

Division

Denition
Let a and b be integers with a = 0. Then we say, a divides b (written as a | b) if there is an integer c such that b = ac. If a | b, we say that a is a factor of b and b is a multiple of a. If a does NOT divide b, we write a b.

The Integers Division Division

Primes and Greatest Common Divisor

Division: some results


Theorem
Let a, b and c are integers. Then if (a | b) and a | c, then a | (b + c) if a | b, then a | bc, for all integers c if a | b and b | c, then a | c.

For (1) only, the rest are for exercise.


Suppose a | b and a | c. Then, by denition of division, there are integers s and t such that b = as and c = at. Hence, b + c = as + at = a(s + t) Therefore, a divides b + c, i.e., a | (b + c).

The Integers Division Division

Primes and Greatest Common Divisor

Division: some results

Theorem
If a,b and c are integers such that a | b and a | c, then a | mb + nc for every integers m and n.

Proof.
Left as exercise. (Hint: use the rst part of the previous theorem).

The Integers Division The Division Algorithm

Primes and Greatest Common Divisor

The division algorithm


Theorem
Let a be an integer and d a positive integer. Then there are unique integers q and r , with 0 r < d such that a = dq + r . The proof of this theorem uses the so-called well-ordering property of nonnegative integers (optional part of this course). In the above theorem, d is called the divisor, a is called the dividend, q is called the quotient, and r is called the remainder. The notation used to express the quotient and remainder: q = a div d, r = a mod d

The Integers Division The Division Algorithm

Primes and Greatest Common Divisor

Example

What are the quotient and remainder when 101 is divided by 11? What are the quotient and remainder when -11 is divided by 3.

The Integers Division The Division Algorithm

Primes and Greatest Common Divisor

Example

What are the quotient and remainder when 101 is divided by 11? What are the quotient and remainder when -11 is divided by 3. By the previous denition, remainder cannot be negative.

The Integers Division The Division Algorithm

Primes and Greatest Common Divisor

Example

What are the quotient and remainder when 101 is divided by 11? What are the quotient and remainder when -11 is divided by 3. By the previous denition, remainder cannot be negative.

Theorem
An integer a is divisible (i.e., can be divided) by an integer d if and only if a mod d = 0

The Integers Division Modular Arithmetic

Primes and Greatest Common Divisor

Modular arithmetic
Denition
Let a and b be integers, and m a positive integer. Then a is congruent to b modulo m written as a b ( mod m) iff m |ab If a and b are not congruent modulo m, we write a b ( mod m)

The Integers Division Modular Arithmetic

Primes and Greatest Common Divisor

Modular arithmetic
Denition
Let a and b be integers, and m a positive integer. Then a is congruent to b modulo m written as a b ( mod m) iff m |ab If a and b are not congruent modulo m, we write a b ( mod m)

Theorem
Let a and b be integers and m a positive integer. Then a b ( mod m) iff a mod m = b mod m Proof of this theorem is left as an exercise.

The Integers Division Modular Arithmetic

Primes and Greatest Common Divisor

Example

Determine whether 17 is congruent to 5 modulo 6. Determine whether 24 and 14 are congruent modulo 6.

The Integers Division Modular Arithmetic

Primes and Greatest Common Divisor

Modular arithmetic and ordinary arithmetic


Theorem
Let m be a positive integer. The integers a and b are congruent modulo m iff there is an integer k such that a = b + km

The Integers Division Modular Arithmetic

Primes and Greatest Common Divisor

Modular arithmetic and ordinary arithmetic


Theorem
Let m be a positive integer. The integers a and b are congruent modulo m iff there is an integer k such that a = b + km

Proof.
If a b ( mod m), then m | (a b). Hence, there is an integer k such that a b = km. This means a = b + km. Conversely, if there is an integer k such that a = b + km, then km = a b. This implies that m divides a b, which means that a b ( mod m). Note: the set of all integers congruent to an integer a modulo m is called the congruence class of a modulo m.

The Integers Division Modular Arithmetic

Primes and Greatest Common Divisor

Addition and multiplication with congruences


Theorem (Addition and multiplication with congruences)
Let m be a positive integer. If a b ( mod m) and c d ( mod m), then a + c b + d ( mod m) and ac bd ( mod m)

The Integers Division Modular Arithmetic

Primes and Greatest Common Divisor

Addition and multiplication with congruences


Theorem (Addition and multiplication with congruences)
Let m be a positive integer. If a b ( mod m) and c d ( mod m), then a + c b + d ( mod m) and ac bd ( mod m)

Proof.
Because a b ( mod m) and c d ( mod m), there are integers s and t such that b = a + sm, and d = c + tm. Hence, b + d = (a + sm) + (c + tm) = (a + c) + (s + t)m bd = (a + sm)(c + tm) = ac + atm + csm + stm2 = ac + m(at + cs + stm) Hence, a + c b + d ( mod m) and ac bd ( mod m)

The Integers Division Modular Arithmetic

Primes and Greatest Common Divisor

Addition and multiplication with congruences


Corollary
Let m be a positive integers and let a and b be integers. Then (a + b) mod m = ((a mod m) + (b mod m)) mod m and ab mod m = ((a mod m)(b mod m)) mod m For the proof, read Rosens book page 205. Note: although the we can do addition and multiplication with congruences, some properties are NOT valid:
if ac bc( mod m), the congruence a b( mod m) may be FALSE. if a b( mod m) and c d( mod m), the congruence ac bd ( mod m) may be FALSE.

The Integers Division Modular Arithmetic

Primes and Greatest Common Divisor

Applications of Congruences

Read Rosens book page 205 208 for more details. Hashing functions: storing records in memory so that retrieval can be done quickly. Pseudorandom numbers: random numbers generated by computers. Cryptology: encryption and decryption of messages, e.g. with Caesar cipher, shift cipher, etc.

The Integers Division Primes

Primes and Greatest Common Divisor

Primes

Denition
A positive integer p > 1 is called prime if the only positive factor of p are 1 and p. A p A positive integer that is greater than 1 and is not prime is called composite Remark: an integer n is composite iff there exists an integer a such that a | n and 1 < a < n. List all primes less than 100

The Integers Division Primes

Primes and Greatest Common Divisor

Fundamental theorem of arithmetic

Theorem (Fundamental theorem of arithmetic)


Every positive integer greater than 1 can be written uniquely as a prime or the product of two or more primes where the prime factors are written in order of nondecreasing size.

The Integers Division Primes

Primes and Greatest Common Divisor

Fundamental theorem of arithmetic

Theorem (Fundamental theorem of arithmetic)


Every positive integer greater than 1 can be written uniquely as a prime or the product of two or more primes where the prime factors are written in order of nondecreasing size. The prime factorization of 100, 641, 999, 1024 are:

The Integers Division Primes

Primes and Greatest Common Divisor

Fundamental theorem of arithmetic

Theorem (Fundamental theorem of arithmetic)


Every positive integer greater than 1 can be written uniquely as a prime or the product of two or more primes where the prime factors are written in order of nondecreasing size. The prime factorization of 100, 641, 999, 1024 are: 100 = 2 2 5 5 = 22 52 .

The Integers Division Primes

Primes and Greatest Common Divisor

Fundamental theorem of arithmetic

Theorem (Fundamental theorem of arithmetic)


Every positive integer greater than 1 can be written uniquely as a prime or the product of two or more primes where the prime factors are written in order of nondecreasing size. The prime factorization of 100, 641, 999, 1024 are: 100 = 2 2 5 5 = 22 52 . 641 = 641

The Integers Division Primes

Primes and Greatest Common Divisor

Fundamental theorem of arithmetic

Theorem (Fundamental theorem of arithmetic)


Every positive integer greater than 1 can be written uniquely as a prime or the product of two or more primes where the prime factors are written in order of nondecreasing size. The prime factorization of 100, 641, 999, 1024 are: 100 = 2 2 5 5 = 22 52 . 641 = 641 999 = 3 3 3 37 = 33 37

The Integers Division Primes

Primes and Greatest Common Divisor

Fundamental theorem of arithmetic

Theorem (Fundamental theorem of arithmetic)


Every positive integer greater than 1 can be written uniquely as a prime or the product of two or more primes where the prime factors are written in order of nondecreasing size. The prime factorization of 100, 641, 999, 1024 are: 100 = 2 2 5 5 = 22 52 . 641 = 641 999 = 3 3 3 37 = 33 37 1024 = 2 2 2 2 2 2 2 2 2 2 = 210

The Integers Division Primes

Primes and Greatest Common Divisor

Prime divisor of a composite integer

Theorem
If n is a composite integer, then n has a prime divisor less than or equal n

The Integers Division Primes

Primes and Greatest Common Divisor

Prime divisor of a composite integer

Theorem
If n is a composite integer, then n has a prime divisor less than or equal n Proof: look at Rosens book page 211. Show that 101 is prime. Find the prime factorization of 7007.

The Integers Division Primes

Primes and Greatest Common Divisor

Innitude of primes

Theorem
There are innitely many primes. Proof: Rosens book page 212. Mersenne primes: a prime that has a special form 2p 1 where p is also a prime.

The Integers Division Primes

Primes and Greatest Common Divisor

Distribution of primes

Theorem (The prime number theorem)


The ratio of number of primes not exceeding x and x/ ln x approaches 1 as x grows without bound, where ln x is the natural logarithm of x.

The Integers Division Greatest Common Divisor and Least Common Multiples

Primes and Greatest Common Divisor

Greatest common divisor

Denition
Let a and b be integers, not both zero. The largest integer d such that d | a and d | b is called the greatest common divisor of a and b (written as gcd(a, b)). What is gcd(24, 36)? What is gcd(17, 22)?

The Integers Division Greatest Common Divisor and Least Common Multiples

Primes and Greatest Common Divisor

Relatively primes
Denition
The integers a and b are relatively prime if gcd(a, b) = 1.

The Integers Division Greatest Common Divisor and Least Common Multiples

Primes and Greatest Common Divisor

Relatively primes
Denition
The integers a and b are relatively prime if gcd(a, b) = 1.

Denition
The integers a1 , a2 , . . . , an are pairwise relatively prime if gcd(ai , aj ) = 1 for every 1 i < j n. Determine whether 10, 17 and 21 are pairwise relatively prime. Determine whether 10, 19 and 24 are pairwise relatively prime.

The Integers Division Greatest Common Divisor and Least Common Multiples

Primes and Greatest Common Divisor

Least common multiple

Denition
The least common multiple of positive integers a and b (written lcm(a, b)) is the smallest positive integer that is divisible by both a and b. What is lcm(23 35 72 , 24 33 )?

Theorem
Let a and b be positive integers. Then ab = gcd(a, b) lcm(a, b)

IKI10600I: Discrete Mathematics I


Integers and Algorithms

Adila A. Krisnadhi
Faculty of Computer Science, University of Indonesia

Representation of Integers

Euclidean Algorithm

Integers and Algorithms


Outline

23

Representation of Integers

24

Euclidean Algorithm

Representation of Integers

Euclidean Algorithm

Theorem (Base b expansion of n)


Let b be a positive integer greater than 1. Then if n is a positive integer, it can be expressed uniquely in the form n = ak bk + ak 1 bk 1 + + a1 b + a0
The base b expansion of n is denoted by (ak ak 1 . . . a1 a0 )b , e.g., (245)8 represents 2 82 + 4 8 + 5 = 165. The base 10 expansion of integers is called decimal expansion. The base 2 expansion of integers is called binary expansion. The base 8 expansion of integers is called octal expansion. The base 16 expansion of integers is called hexadecimal expansion (using 0,1,2,3,4,5,6,7,8,9,A,B,C,D,E, and F as digits where A through F correspond to numbers 10 through 15 in decimal).

Representation of Integers

Euclidean Algorithm

Base b expansion to decimal expansion

(101011111)2 = 1 28 + 0 27 + 1 26 + 0 25 + 1 24 + 1 23 + 1 22 + 1 21 + +1 20 = (351)10 (2AE0B)16 = 2164 +10163 +14162 +016+11 = (175627)10 (3071)8 = 3 83 + 0 82 + 7 8 + 1 = (1593)10

Representation of Integers

Euclidean Algorithm

Decimal expansion to base b expansion


Find octal expansion of 12345

Representation of Integers

Euclidean Algorithm

Decimal expansion to base b expansion


Find octal expansion of 12345 Take the remainders of the following divisions: 12345 = 8 1543 + 1 1543 = 8 192 + 7 192 = 8 24 + 0 24 = 8 3 + 0 3=80+3 Hence, (12345)10 = (30071)8 Find hexadecimal expansion of (177130)10 . Find binary expansion of (241)10 .

Representation of Integers

Euclidean Algorithm

Finding GCD with Euclidean algorithm

Find gcd(91, 287).

Representation of Integers

Euclidean Algorithm

Finding GCD with Euclidean algorithm

Find gcd(91, 287).

287 = 91 3 + 14 91 = 14 6 + 7 14 = 7 2 + 0 Hence, gcd(91, 287) = 7

Representation of Integers

Euclidean Algorithm

Euclidean algorithm

procedure : gcd ( a , b : p o s i t i v e i n t e g e r s ) x := a y := b while y = 0 begin r := x mod y x := y y := r end { gcd(a, b) i s x }

Representation of Integers

Euclidean Algorithm

Euclidean algorithm: Why it works?

Theorem
Let a = bq + r , where a, b, q, and r are integers. Then gcd(a, b) = gcd(b, r ). For example, in the previous slide, it holds that: gcd(91, 287) = gcd(91, 14) = gcd(14, 7) = gcd(7, 0) = 7.

You might also like